First Week Drop In Schedule

IOD Incorporated
May 2013
CCS Preparation Readiness
W k h
Workshop
May 2013
Presented by:
Kim Felix, RHIA, CCS
Director of Education
IOD Incorporated
www.iodincorporated.com
• Full suite of HIM solutions including:
–
–
–
–
–
–
Release of Information (ROI)
Document Conversion
Coding/Abstracting/Auditing
ICD-10 Consulting
g
RAC Services
Training/Education
www.iodincorporated.com
2
CCS Prep Workshop
Kim Felix, RHIA, CCS
AHIMA Approved ICD-10 Trainer
1
IOD Incorporated
May 2013
Where to begin?
• Organize and review the following:
– AHIMA Candidate Handbook for the CCS Exam
– Official Coding Guidelines (inpt and outpt)
– Coding books
– Coding Clinic from the past 2-3 years (make
notes in ICD-9 book)
– CPT Assistant from the past 2-3 years
– Personal textbooks, class notes, tests, etc.
3
General Information
• Inpatient rules versus Outpatient rules
– i.e. possible/probable
• Do not overcode!
– signs/symptoms integral to a disease process
– Personal and family �history of’ codes that have no
bearing on the stay
• Pharmacology
– what drugs treat what condition?
• When to query?
– abnormal lab findings not correlated by the
physician in the record
• Reimbursement and Billing
4
CCS Prep Workshop
Kim Felix, RHIA, CCS
AHIMA Approved ICD-10 Trainer
2
IOD Incorporated
May 2013
General Information
• 2013 CCS exam will launch on June 1,
2013 Candidates
2013.
C did t will
ill b
be required
i d tto b
bring
i
the 2012 OR 2013 versions of ICD-9-CM
Volumes 1 through 3 and the 2013 AMA
CPT coding books to the testing center
– Per AHIMA’s website
– You can use your code books for the entire
exam
5
Exam Specifications
•
•
•
•
•
Total Testing Time= Four HOURS
Multiple Choice Section
– 81 single response multiple-choice items (63 "scored" and 18 "pre-test"
items)
– Pre
Pre-test
test items are unscored items that are included in the examination to
assess the item's performance prior to using it for operational use in a
future examination
– The pre-test items are scrambled randomly throughout the examination
and do not count toward the candidate's score.
Multiple Select Section – more than one response required
– The multiple select section will consist of 8 multiple response items (6
"scored" and 2 "pre-test" items). Pre-test items are unscored items.
Fill in the Blank Section (Medical Record Cases) - The fill in the blank section
will consist of 12 medical record cases, which contains six outpatient records
and six inpatient records.
Inpatient diagnoses and procedures are to be coded with ICD-9-CM volumes
1-3; ambulatory care diagnoses are to be coded with ICD-9-CM volumes 1
and 2; and ambulatory care procedures with CPT.
6
CCS Prep Workshop
Kim Felix, RHIA, CCS
AHIMA Approved ICD-10 Trainer
3
IOD Incorporated
May 2013
Exam Specifications
• Candidates will be instructed on exactly how
many codes are required
– By diagnosis and procedure
• If a candidate
did t d
does nott enter
t an answer in
i one off
the required boxes, they will NOT be allowed to
move to the next question
• Scoring:
– If the question has 2 answers and the candidate gets
both codes correct, they will receive 2 points
– If they get one correct and one incorrect, they will
receive 1 point
– Not �all or nothing’ questions.
– Partial credit is given.
7
What to expect on the test
TOPIC
% of questions
Health information documentation
10
Diagnosis and procedure coding
64
Regulatory Guidelines & Reporting
Requirements for Inpatient
5
Regulatory Guidelines & Reporting
Requirements for Outpatient
6
Data Quality
4
Information and Communication Technologies
3
Privacy, Confidentiality, Legal and Ethical Issues
4
Compliance
4
8
CCS Prep Workshop
Kim Felix, RHIA, CCS
AHIMA Approved ICD-10 Trainer
4
IOD Incorporated
May 2013
BOOKS
• http://www.ahima.org/certification
• Candidate Exam available
– Candidates who report to the test center
with incorrect code books will not be
permitted to test and will forfeit their exam
application fee.
9
Inpatient
p
Guidelines
10
CCS Prep Workshop
Kim Felix, RHIA, CCS
AHIMA Approved ICD-10 Trainer
5
IOD Incorporated
May 2013
Selection of Principal Diagnosis
• Codes for signs/symptoms are not to be used
as principal dx when a related definitive
diagnosis has been established
• When
Wh th
there are 2 or more iinterrelated
t
l t d
conditions that potentially meet the definition
of principal diagnosis, either condition may be
sequenced first
• When there are 2 or more diagnoses that
equally
q
y meet the criteria for p
principal
p
diagnosis, any one of the diagnoses may be
sequenced first
11
Selection of Principal Diagnosis
• When 2 or more contrasting or comparative
diagnoses are documented as �either/or’, they are
coded as if they were confirmed and sequenced
according to the circumstances of admission
• When a symptom is followed by contrasting/
comparing (either/or) diagnoses, the symptom code
is sequenced first followed by all the contrasting/
comparative diagnoses
• Sequence the condition that meets the definition of
principal diagnosis even though treatment may not
have been carried out due to unforeseen
circumstances
12
CCS Prep Workshop
Kim Felix, RHIA, CCS
AHIMA Approved ICD-10 Trainer
6
IOD Incorporated
May 2013
Selection of Principal Diagnosis
• When the admission is for treatment of a
complication resulting from surgery or other medical
care,, the complication
p
code is sequenced
q
as the
principal diagnosis. Add an additional code to
further describe the complication if the complication
code lacks specificity
• If the diagnosis is qualified as �probable, suspected,
likely, questionable, possible, still to be ruled out’ or
other similar terms indicating uncertainty, code the
condition as if it existed or was established
13
Sample Question
• The coding supervisor conducts weekly quality controls to
assess the accuracy of coded data. Which of the following
codes listed as the principal diagnosis is the only code
appropriate for principal diagnosis assignment?
A. 321.2 Meningitis due to viruses, not
elsewhere classified
B. E855.0 Accidental poisoning by anticonvulsant and
anti-Parkinsonism drugs
C. V27.0 Outcome of delivery, single liveborn
D. V71.1 Observation for suspected malignant
neoplasm
14
CCS Prep Workshop
Kim Felix, RHIA, CCS
AHIMA Approved ICD-10 Trainer
7
IOD Incorporated
May 2013
Answer
пЃ± D. V71.1 Observation for suspected
malignant neoplasm
15
Additional Diagnoses
•
•
•
•
•
•
•
Clinical evaluation
Therapeutic treatment
Diagnostic procedures
Extended LOS
Increased nursing care/monitoring
Previous conditions, if pertinent to current stay
Abnormal findings, if provider documents their
clinical significance
• Uncertain diagnoses (probable, suspected, likely,
questionable, possible, still to be ruled out) are
coded as if they existed or established
16
CCS Prep Workshop
Kim Felix, RHIA, CCS
AHIMA Approved ICD-10 Trainer
8
IOD Incorporated
May 2013
Additional Diagnoses
• AHIMA’s website: Procedures for Coding Part II
of the CCS Exam
• Highlights:
– Code diagnoses of chronic systemic or generalized
conditions that are not under active management
when a physician documents them in the record and
that may have a bearing on the management of the
patient.
• For example: Admission for breast mass; diagnosis is
carcinoma. Patient is blind and requires increased care.
Code the breast carcinoma and blindness.
Additional Diagnoses
• More highlights:
– Code status post previous surgeries or conditions
likely to recur that may have a bearing on the
managementt off the
th patient
ti t
• For example: Admission for pneumonia; status post cardiac
bypass surgery. Code the pneumonia and status post cardiac
bypass surgery (V code).
– Do not code status post previous surgeries or
histories of conditions that have no bearing on the
management of the patient.
• For example: Admission for pneumonia; status post hernia
repair six months prior to admission. Code only the
pneumonia.
• Previous surgeries involving transplants, internal
devices, and prosthetics should be coded.
CCS Prep Workshop
Kim Felix, RHIA, CCS
AHIMA Approved ICD-10 Trainer
9
IOD Incorporated
May 2013
Additional Diagnoses
• More highlights:
• Do not assign External Cause of Injury
and Poisoning Codes
C
((E codes),
) except
those that identify the causative substance
for an adverse effect of a drug that is
correctly prescribed and properly
administered and/or p
poisoning
g ((E850E982).
Procedures
• Do not code procedures that fall within the code range
87.01 through 99.99. But code procedures in the
following ranges:
–
–
–
–
–
–
–
–
–
–
–
87.51 87.54 Cholangiograms
87.51-87.54
87.74 and 87.76 Retrogrades, urinary systems
88.40-88.58 Arteriography and angiography
92.21-92.29 Radiation therapy
94.24-94.27 Psychiatric therapy
94.61-94.69 Alcohol/drug detoxification and rehabilitation.
96.04 Insertion of endotracheal tube
96 56 Other lavage of bronchus and trachea
96.56
96.70-96.72 Mechanical ventilation
98.51-98.59 ESWL
99.25 Chemotherapy
CCS Prep Workshop
Kim Felix, RHIA, CCS
AHIMA Approved ICD-10 Trainer
10
IOD Incorporated
May 2013
Coding from Pathology/Lab/Radiology
Reports
• In the inpatient setting, coders should not
assign
g codes from the p
pathology
gy report
p
alone, without the provider’s input
• When available, the coder may use x-ray
results to provide greater specificity
• Abnormal findings alone should not be
coded
d d unless
l
th
the provider
id d
documents
t th
their
i
clinical significance
21
Coding abnormal finding from the
pathology report
• Abnormal findings on the pathology report are not coded
and reported unless the provider indicates their clinical
significance. This ensures that the documentation and the
codes reported are consistent with the attending physician’s
interpretation since he or she is responsible for the clinical
management of the case. It is the responsibility of the
attending physician to gather and collate all of the findings
from the consultants and other providers involved in the care
for the patient. The plan of care is based on the attending’s
evaluation, interpretation and collation of all the findings (i.e..
pathology radiology,
pathology,
radiology and laboratory results)
results). Although the
pathologist provides a written interpretation of a tissue biopsy,
this is not equivalent to the attending physician’s medical
diagnosis based on the patient’s complete clinical picture.
Coding Clinic 2008
22
CCS Prep Workshop
Kim Felix, RHIA, CCS
AHIMA Approved ICD-10 Trainer
11
IOD Incorporated
May 2013
Infectious and Parasitic Diseases
• HIV
– Code only confirmed cases
– V08 = asymptomatic
y p
HIV infection who have never had
an HIV-related illness
– HIV-related condition = Code 042 as principal
– Previous diagnosis of AIDS/HIV related illness is always
coded as 042 from then on
– PREGNANCY is the exception- Use code from
pregnancy chapter as principal diagnosis if pregnant
woman has HIV-related illness
– Patient here to determine HIV status = V73.89
23
Infectious and Parasitic Diseases
• Septicemia, Systemic Inflammatory
Response Syndrome (SIRS), Sepsis,
Severe Sepsis
Sepsis, and Septic Shock
Bacteremia→→→Septicemia→→→Sepsis↓
↓Severe Sepsis w/shock←←Severe Sepsis←←
в†’в†’Multiple
p Organ
g Dysfunction→→Death
y
24
CCS Prep Workshop
Kim Felix, RHIA, CCS
AHIMA Approved ICD-10 Trainer
12
IOD Incorporated
May 2013
Sepsis/SIRS
• Bacteremia- bacteria in the blood (790.7)
• Septicemia- systemic disease with presence of
organisms in the blood (038.x)
– Do NOT need a positive blood culture to code
septicemia
• SIRS – systemic response to infection, trauma/burns,
cancer WITH symptoms (fever, tachycardia,
tachypnea, leukocytosis) (995.90)
progresses quickly to…..
• Sepsis – SIRS due to infection (995
(995.91)
91)
• Severe sepsis – sepsis with associated acute organ
dysfunction (995.92)
25
Bottom Line…
• Coding of SIRS, sepsis and severe sepsis requires a
minimum of 2 codes:
– One for underlying cause (infection, trauma)
Thi iis sequenced
This
d fi
firstt
– One for SIRS code (995.9x) as secondary dx
– Severe sepsis also requires a code for the associated
organ dysfunction
EXCEPTION:
If the patient has sepsis and an acute organ
g
dysfunction, but the documentation indicates that the
acute organ dysfunction is related to a medical
condition other than sepsis, do not assign code 995.92
26
CCS Prep Workshop
Kim Felix, RHIA, CCS
AHIMA Approved ICD-10 Trainer
13
IOD Incorporated
May 2013
Septic Shock
• Code first the systemic infection, followed
by 995.92 and 785.52
• Septic
S ti Shock
Sh k cannott b
be assigned
i
d as
principal diagnosis
• Must use code 995.92 with 785.52 even if
the term �severe sepsis’ is not documented
27
Sepsis and Severe Sepsis with
Non-infectious process
• SIRS can develop due to trauma,
malignant neoplasm or pancreatitis
• If no infection is documented as the cause
of SIRS, code the underlying condition
(such as the injury) followed by 995.93 or
995.94.
• Additional
Additi
l codes
d ffor organ d
dysfunction
f
ti
should also be assigned as secondary
diagnosis
28
28
CCS Prep Workshop
Kim Felix, RHIA, CCS
AHIMA Approved ICD-10 Trainer
14
IOD Incorporated
May 2013
Sepsis Example
• Septicemia due to methicillin-resistant
Staph A. Patient was also admitted with
septic shock and Stage 3 decubitus ulcer
of the sacrum. Patient had a central line
inserted in the ICU for medication
administration.
038.12, 995.92, 785.52, 707.03, 707.23
38 93
38.93
29
Sample Question
• The following ICD-9-CM index entries appear:
Encephalitis
infectious (acute) (virus) NEC 049.8
postinfectious NEC 136.9
136 9 [323.6]
[323 6]
The diagnosis listed by the physician is encephalitis
after infection. Which of the following represents the
correct coding and sequencing?
A. 049.8
B. 323.6
C. 136.9; 323.6
D. 049.8; 136.9
30
CCS Prep Workshop
Kim Felix, RHIA, CCS
AHIMA Approved ICD-10 Trainer
15
IOD Incorporated
May 2013
Answer
•
C. 136.9; 323.6
31
Neoplasms
• Treatment directed at malignancy= code malignancy as
principal diagnosis
• Treatment directed at complication associated with
malignancy (i.e. anemia)
anemia)= code associated complication
as principal diagnosis
• Primary malignancy previously excised and NO further
treatment = V code
– There is no code for a history of metastatic site(s)
• Admission for chemotherapy/radiation = V58.1x as
principal diagnosis, then code malignancy as secondary
• Signs/symptoms associated with or related to the
malignancy= use malignancy as principal diagnosis
32
CCS Prep Workshop
Kim Felix, RHIA, CCS
AHIMA Approved ICD-10 Trainer
16
IOD Incorporated
May 2013
Neoplasms
• Question: Patient admitted with weakness,
fatigue and weight loss. Hepatomegaly and
jaundice are found on PE. CT is compatible with
extensive
t
i metastatic
t t ti di
disease iinvolving
l i th
the llungs,
liver and intra-abdominal lymph nodes. Primary
tumor was not identified although the possibility of
a pancreatic mass was considered. Discharge
diagnoses included “the liver and lung masses,
probably metastatic disease”. How should this be
coded?
33
Neoplasms
• ANSWER: Assign code 197.0, secondary neoplasm of
lung and 197.7, secondary neoplasm of liver and 199.1,
malignant neoplasm, unspecified site. In this case the
Official Guideline for Coding and Reporting Uncertain
Diagnoses would apply.
• The Cooperating parties for ICD-9-CM have thoroughly
examined this issue and decided NOT to change or
make an exception to the existing guideline for uncertain
diagnoses. If the diagnosis documented at the time of
discharge (inpatient) is qualified as �probable
probable,
suspected, likely, possible or still to be ruled out’, code
the condition as if it existed or was established.
– Coding Clinic, 2006
34
CCS Prep Workshop
Kim Felix, RHIA, CCS
AHIMA Approved ICD-10 Trainer
17
IOD Incorporated
May 2013
Neoplasms
• Tumor lysis syndrome
• Effective October 1, 2009, subcategory 277.8, Other specified
disorders of metabolism, has been expanded to uniquely describe
tumor lysis syndrome. This new code (277.88) captures both tumor
y syndrome
y
following
g cancer therapy
py and spontaneous
p
tumor lysis
y
lysis
syndrome. Previously, the tumor lysis syndrome was not specifically
indexed in ICD-9-CM.
• Tumor lysis syndrome (TLS) refers to a group of serious, potentially
lifethreatening metabolic disturbances that can occur after
antineoplastic therapy. TLS usually occurs following the administration
of anti-cancer drugs; however, it can also develop spontaneously or as
a result of radiation or corticosteroid therapy. It is often associated with
leukemias and lymphomas, but is also seen in other hematologic
malignancies and solid tumors. Since anti-cancer therapy can result in
the quick breakdown of tumor cells, some malignancies with rapidly
dividing cells that are very responsive to therapy are at an increased
risk for TLS. When cancer cells are destroyed, they can release
intracellular ions and metabolic byproducts into the circulation
leading to TLS.
35
Neoplasms
• Tumor lysis syndrome
• Clinically, the syndrome is characterized by a number of effects
from tumor cell breakdown and as phosphate levels increase,
serum calcium decreases; if the levels are both high, calcium
phosphate may precipitate in tissues
tissues.
– These disturbances can lead to acute renal failure due to uric
acid nephropathy and/or xanthine nephropathy, or due to
precipitation of calcium phosphate in renal tubules or
interstitium.
• Although TLS is commonly a complication of cancer therapy, the
syndrome may rarely develop spontaneously before treatment is
initiated. Pretreatment spontaneous tumor lysis syndrome is
generally associated with acute renal failure due to uric acid
nephropathy.
• An additional E code should be assigned to identify the cause
when tumor lysis syndrome is drug-induced.
36
CCS Prep Workshop
Kim Felix, RHIA, CCS
AHIMA Approved ICD-10 Trainer
18
IOD Incorporated
May 2013
Neoplasm examples
• Patient with h/o malignant neoplasm of lung is
admitted with seizures. W/U revealed mets to
the brain.
198.3, 780.39, v10.11
• Patient admitted for dehydration following
chemotherapy as treatment for ovarian CA
276.51, 183.0
37
Neoplasms
• Question: The patient is a 48-year-old male with
glioblastoma multiforme status post two
surgeries. The tumor has recurred with massive
growth since debulking one month ago. The
provider indicated that there was a significant
amount of surrounding vasogenic edema and
mass effect. Is it appropriate to assign a code for
cerebral edema when it is due to a primary
intracranial process such as a brain tumor and
the provider has indicated that it is clinically
significant?
38
CCS Prep Workshop
Kim Felix, RHIA, CCS
AHIMA Approved ICD-10 Trainer
19
IOD Incorporated
May 2013
Neoplasms
• Answer: Assign code 348.5, Cerebral edema, as
an additional diagnosis, since the provider has
evaluated and documented the clinical significance
of the vasogenic edema.
• Patients with glioblastoma commonly develop vasogenic
edema. Vasogenic edema is an accumulation of fluid in the
brain (due to the tumor’s disruption of the blood-brain
barrier). The surrounding edema can increase the mass
effect of the tumor and is considered an irreversible process.
Tumor-related vasogenic edema may disrupt synaptic
transmission and alter neuronal excitability, leading to
headaches, seizures, focal neurological
g
deficits, and
encephalopathy. The condition can contribute to morbidity,
resulting in fatal brain herniation.
– Coding Clinic, Third Quarter 2009
39
Neoplasm examples
• Pt admitted with abdominal pain. Needle
biopsy of liver reveals secondary
malignancy of the liver
liver. Pt has an
exploratory laparotomy to determine
primary site. Primary site is unknown at
time of discharge.
197.7, 199.1, 54.11, 50.11
40
CCS Prep Workshop
Kim Felix, RHIA, CCS
AHIMA Approved ICD-10 Trainer
20
IOD Incorporated
May 2013
Neoplasms
• Malignant Pleural Effusion
– Code 511.8, other effusion has been further
expanded to differentiate between malignant
pleural effusion (511.81) and other specified
forms of pleural effusion (511.89)
– Previously malignant pleural effusion
defaulted to secondary neoplasm of pleura
(197 2)
(197.2)
41
Neoplasms
• Question: The patient was admitted for heminephrectomy due to
bilateral renal masses. Pathologic analysis confirmed renal cell
carcinoma. The provider listed “bilateral renal masses,” in the final
diagnostic statement since the pathological results were not
available at the time. However, the cancer staging form that the
provider has completed and signed is available in the health record.
Our medical staff leadership has deemed this confirmation of the
pathologic diagnosis of renal cancer and sufficient documentation
for coding. Is the completed and signed cancer staging form
appropriate documentation for coding and reporting purposes?
• Answer: Yes, it is appropriate to use the completed cancer staging
form for coding purposes when it is authenticated by the attending
physician.
p
y
As stated in Coding
g Clinic, Second Quarter 2000, p
pages
g
17- 18, “Documentation is not limited to the face sheet, discharge
summary, progress notes, history and physical, or other report
designed to capture diagnostic information. This advice refers only
to inpatient coding.”
Coding Clinic, 2nd quarter 2010, p. 7-8
42
CCS Prep Workshop
Kim Felix, RHIA, CCS
AHIMA Approved ICD-10 Trainer
21
IOD Incorporated
May 2013
Sample Question
• A patient is admitted to the hospital to undergo a radical
mastectomy for recurrent carcinoma of the breast (Previously,
she had elected to have a lumpectomy). The attending
physician lists a history of atrial fibrillation as a secondary
di
diagnosis.
i Th
The patient
ti t iis currently
tl nott on any medication.
di ti
F
For
medical clearance prior to surgery, the patient is seen by a
consultant, who says that the patient was satisfactory for the
procedure. The coder should:
A. Report atrial fibrillation as a current condition, because it was
documented by the physician in the history.
B. Code atrial fibrillation as a current condition, because the patient was
seen by a consultant for surgical clearance.
C Add th
C.
the code
d ffor observation
b
ti ffor suspected
t d cardiovascular
di
l condition
diti
D. Omit reporting the code for atrial fibrillation, because it
was not treated and did not affect the course of treatment.
43
Answer
пЃ± D. Omit reporting the code for atrial
fibrillation because it was not treated and
fibrillation,
did not affect the course of treatment.
44
CCS Prep Workshop
Kim Felix, RHIA, CCS
AHIMA Approved ICD-10 Trainer
22
IOD Incorporated
May 2013
Diabetes
• Type I versus Type II
– Type I is always insulin-requiring (usually a
jjuvenile onset)) and is much less common
than Type II
– Type II CAN be insulin- requiring or can be
managed with medication. Add v58.67 for
patients that are Type II and who routinely
use insulin
– If the type of DM is not documented in the
medical record the default is type II
45
Diabetes
• Fifth digits must be consistent for Diabetes
coding.
• If DM is out of control,
control all manifestation codes
must have the same fifth digit of uncontrolled.
– Example: Patient has Type II Diabetes with
numbness due to peripheral neuropathy. Patient
also has retinopathy. Patient becomes
uncontrolled on Day 3.
Final Dx: 250.62, 357.2, 250.52, 362.01
46
CCS Prep Workshop
Kim Felix, RHIA, CCS
AHIMA Approved ICD-10 Trainer
23
IOD Incorporated
May 2013
Diabetes
• Question: Previously published Coding Clinic
advice instructed coders not to assume a causal
relationship between diabetes and certain
manifestations The physician documentation
manifestations.
should establish the relationship with terms such
as “due to diabetes” or “diabetic.” However,
recent advice published in Coding Clinic Third
Quarter 2008, pages 5-6, is not consistent with
this advice. Additionally, in ICD-9- CM’s
Alphabetic Index, there is no entry for diabetes
with
ith neuropathy.
th C
Could
ld you please
l
clarify
l if thi
this
new instruction?
47
Diabetes
• Answer: In ICD-9-CM’s Alphabetic Index,
the subentry term “with” means associated
with or due to.
to If the provider documents
“diabetes with neuropathy,” assign code
250.6X, Diabetes with neurological
manifestations, and code 357.2,
Polyneuropathy in diabetes.
• Coding Clinic, Second Quarter 2009
48
CCS Prep Workshop
Kim Felix, RHIA, CCS
AHIMA Approved ICD-10 Trainer
24
IOD Incorporated
May 2013
Sample Question
• A patient with out-of-control •
Type I Diabetes was
admitted with diabetic
retinopathy.
ti
th Th
The correctt
coding and sequencing for
•
this case is…
Select ONE of the following
options
a. 250.53, 362.02
•
b. 250.53, 362.01
c. 250.51, 362.01
•
d. 250.51, 362.02
250.51- Type I DM with
ophthalmic
manifestations,, not
stated as uncontrolled
250.53- Type I DM with
ophthalmic
manifestations,
uncontrolled
362 01 Background
362.01diabetic retinopathy
362.02- Proliferative
diabetic retinopathy
Answer
пЃ± b. 250.53, 362.01
250.53 Type I DM, OOC
362 01 Background diabetic retinopathy
362.01
Diabetes
with
retinopathy 250.5 [362.01]
background
d e to secondary
due
secondar diabetes
due to secondary diabetes
nonproliferative
CCS Prep Workshop
Kim Felix, RHIA, CCS
AHIMA Approved ICD-10 Trainer
25
IOD Incorporated
May 2013
Diabetes
• How should �brittle’ diabetes be coded?
– 250.0x, DM without mention of complication.
Brittle is a nonessential modifier under
diabetes. Code 250.90 is NOT an
appropriate code assignment.
• Coding Clinic, 2006
• Review the record for documentation of �out of
control’ diabetes
51
Diabetes
• Question: How should a diagnosis of borderline diabetes
be coded?
• Answer: Code assignment for borderline diabetes mellitus
should be based on physician documentation and will
require clarification in some cases. If the attending
physician has confirmed a diagnosis of diabetes mellitus,
assign the appropriate code from category 250, Diabetes
mellitus. Otherwise, a diagnosis of “borderline diabetes”
without further confirmation of the disease should be
assigned the appropriate code from subcategory 790.2,
Ab
Abnormal
l glucose.
l
Coding Clinic, First Quarter, 2011
52
CCS Prep Workshop
Kim Felix, RHIA, CCS
AHIMA Approved ICD-10 Trainer
26
IOD Incorporated
May 2013
Diabetes
Question: How is diabetes with ketoacidosis
coded?
Answer: Assign code 250.13, Diabetes with
ketoacidosis type I [juvenile type]
ketoacidosis,
type],
uncontrolled, for diabetic ketoacidosis.
Diabetic ketoacidosis by definition is
uncontrolled and code 250.13 is the default,
unless the MD specifically documents type II.
• Coding Clinic, 2006
• This advice is inconsistent with our �normal’
normal diabetes
guidelines that states Type II is the default!
53
Sample Question
• A patient was admitted to the hospital with severe
dehydration and malnutrition. His blood sugar was
elevated. The patient is a known alcohol abuser.
Intravenous fluid replacement was given to hydrate the
patient, who signed out against medical advice after two
days. Final diagnoses were: severe dehydration with
malnutrition and adult-onset diabetes vs. early cirrhosis
associated with alcoholism. The principal diagnosis is:
A. Adult-onset diabetes
B. Alcohol abuse
C. Cirrhosis of liver due to alcoholism
D. Severe dehydration
54
CCS Prep Workshop
Kim Felix, RHIA, CCS
AHIMA Approved ICD-10 Trainer
27
IOD Incorporated
May 2013
Answer
•
D. Severe dehydration
55
Insulin Pump
• Any insulin pump malfunction resulting
in overdose or underdose of insulin=
mechanical complication due to insulin
pump (996.57). If overdose, add 962.3 as
secondary diagnosis for the poisoning by
insulin and the correct DM code as well.
56
CCS Prep Workshop
Kim Felix, RHIA, CCS
AHIMA Approved ICD-10 Trainer
28
IOD Incorporated
May 2013
Blood and Blood forming organs
• Use of 286.5
– Do not use 286.5 if patient has a
h
hemorrhagic
h i di
disorder
d and
d iis on an
anticoagulant drug.
– 286.5 is very rarely used and is an
inherent condition of clotting factors, not
one that is a result of drug
g usage.
g
57
Anemia
• Anemia of chronic disease
– 285.2x can be used as principal if the
encounter is for the treatment of anemia
– Code also the chronic condition causing
the anemia
• 285.21- anemia in chronic kidney disease
• 285.22- anemia in neoplastic disease
• 285.29- anemia in other chronic illness
58
CCS Prep Workshop
Kim Felix, RHIA, CCS
AHIMA Approved ICD-10 Trainer
29
IOD Incorporated
May 2013
Anemia
• Question: The provider’s final diagnostic
statement listed “chronic anemia.” Should code
285.9, Anemia, unspecified, or code 281.9,
U
Unspecified
ifi d d
deficiency
fi i
anemia,
i b
be assigned
i
d since
i
code 281.9 includes “chronic simple anemia”?
• Answer: Assign code 285.9, Anemia, unspecified,
for chronic anemia not otherwise specified.
Coding Clinic, First Quarter 2011
59
Mental Disorders
• Abuse versus Dependence
– Abuse = take drugs to excess but has not yet
reached a state of dependence
• 305.x
– Dependence = chronic condition of patient
developing a pattern of drug abuse with
increased tolerance and is unable to stop using
the drug, even while health, social interactions
and jjob p
performance are impaired
p
• 303.xx and 304.xx
60
CCS Prep Workshop
Kim Felix, RHIA, CCS
AHIMA Approved ICD-10 Trainer
30
IOD Incorporated
May 2013
Mental Disorders
• 5th digits associated with abuse and
dependence
– 1 ContinuousContinuous Daily (or almost daily) use of
drug/alcohol
– 2 Episodic- �Binging’ lasting weeks or months
followed by no use
– 3 Remission- Complete cessation of
drug/alcohol or period which decrease
towards cessation is taking place
61
Mental Disorders
• Be aware of current status of condition
– i.e. acute, chronic, exacerbations, with/without
psychosis
– Bipolar d/o, mixed, currently depressed with moderate
stress
Final dx: 296.50- bipolar I disorder, most recent
episode (or current) depressed, unspecified
Coding Clinic, 2006
62
CCS Prep Workshop
Kim Felix, RHIA, CCS
AHIMA Approved ICD-10 Trainer
31
IOD Incorporated
May 2013
Mental Disorders
• Question: Which code should we use to capture patients who
are heroin addicts and are being maintained on
methadone? Coding Clinic Fourth Quarter 1988, page 8,
advised coders to use code 304.01 for patients who are
receiving methadone maintenance; however
however, code V58
V58.69
69
appears more appropriate.
• Answer: Assign code 304.00, Opioid type dependence,
unspecified, for patients who are receiving methadone
maintenance because of heroin dependence. Code V58.69,
Long-term (current) use of other medications, is not appropriate
since it should not be used for patients who have addictions to
drugs. The Official Guidelines for Coding and Reporting state
that subcategory V58.6
V58 6 is not used for medications for
detoxification or maintenance programs to prevent withdrawal
symptoms in patients with drug dependence (e.g., methadone
maintenance for opiate dependence).
Coding Clinic, 2nd quarter 2010, p. 13
63
Nervous System
• Clarification of �Pain’ Codes
– If the pain is not specified as �acute’ or �chronic’
do not assign codes from 338.x, except for postthoracotomy pain
pain, post- op pain,
pain neoplasm pain
or central pain syndrome
– These include:
•
•
•
•
•
Central pain syndrome
Acute pain
Chronic pain
Neoplasm related pain (acute or chronic)
Chronic pain syndrome
64
CCS Prep Workshop
Kim Felix, RHIA, CCS
AHIMA Approved ICD-10 Trainer
32
IOD Incorporated
May 2013
Nervous System
• 338.1x- Acute pain
• 338.2x- Chronic pain
• There is no time frame defining when pain
becomes chronic. Provider documentation
should define the pain as chronic
65
Nervous System
• 338 codes as Principal Diagnosis
– When pain control or pain management is
the reason for admission
• Code the underlying cause of the pain as
an additional diagnosis, if known.
– Insertion for neurostimulator for pain
control
66
CCS Prep Workshop
Kim Felix, RHIA, CCS
AHIMA Approved ICD-10 Trainer
33
IOD Incorporated
May 2013
Nervous System
• When NOT to use 338 codes as
principal
– Admission is for a procedure aimed at
treating the underlying condition
• Code the condition as principal diagnosis
67
Nervous System
• Neoplasm Related Pain
– 338.3 is assigned when pain is related to or
associated with anyy type
yp of cancer, regardless
g
if
the pain is documented as acute or chronic
– 338.3 may be assigned as principal when the
patient is admitted for pain control purposes.
Code the neoplasm as a secondary diagnosis.
– �Chronic pain syndrome’ is different than
�chronic pain’.
68
CCS Prep Workshop
Kim Felix, RHIA, CCS
AHIMA Approved ICD-10 Trainer
34
IOD Incorporated
May 2013
Nervous System
• Accidental Dural Tear
– New codes have been created for intraoperative
incidental/inadvertent dural tear (349.31) and other dural
tear (349.39) in order to distinguish dural tears from
other
h types off accidental
id
l surgical
i l llacerations.
i
Th
The d
dura
mater covers the spinal cord and spinal nerves. A tear
in the dura that occurs during spinal surgery is not
unusual and is typically repaired intra-operatively when
identified. Primary closure of the dural tear is usually
accomplished. Dural tears that are not discovered
during surgery can result in leakage of CSF, leading to
CSF headache,, caudal displacement
p
of the brain,,
subdural hematoma, spinal meningitis,
pseudomeningocele and/or a dural cutaneous fistula.
– Coding Clinic 2008
69
Sample Question
• A patient was admitted with complaints of severe
vertigo, headache, and nausea of two weeks’ duration.
The patient had a malignant melanoma of the face
removed two y
years ago.
g An MRI ordered during
g this
stay showed no sign of malignancy; however, toxicology
studies indicated high levels of insecticide in the blood,
which the physician documented as being toxic
neuropathy.
• Which of the following represents the conditions to be
coded?
A. Toxic neuropathyy due to insecticide
B. Toxic neuropathy due to insecticide; history of
malignant melanoma
C. Vertigo; headaches; nausea; malignant melanoma
D. Vertigo; headaches; nausea; insecticide poisoning
70
CCS Prep Workshop
Kim Felix, RHIA, CCS
AHIMA Approved ICD-10 Trainer
35
IOD Incorporated
May 2013
Answer
•
B. Toxic neuropathy due to
insecticide;history of malignant
melanoma
71
Circulatory System
• Hypertension
• Presume a cause and effect relationship with
HTN and Chronic Kidney Disease (CKD) unless
it is documented that the CKD is due to another
cause (i.e. DM)
– Use 403.xx for Hypertensive CKD
– There is NO cause and effect relationship with HTN
and Acute Renal Failure
• Use 2 codes 584.9 and 401.9
72
CCS Prep Workshop
Kim Felix, RHIA, CCS
AHIMA Approved ICD-10 Trainer
36
IOD Incorporated
May 2013
Circulatory System
• Hypertension
– Do NOT presume cause and effect
relationship
l ti
hi with
ith HTN and
dh
heartt di
disease.
Must be documented as �hypertensive’ or �due
to’
– Use additional code from category 428 to
identify the type of heart failure, if
documented
73
Circulatory System
• Hypertension
– Uncontrolled HTN≠Malignant/Accelerated HTN
– There is no code for uncontrolled
ncontrolled
hypertension. Do NOT assume this is
malignant.
– Elevated BP or transient hypertension without
a diagnosis of HTN = 796.2
74
CCS Prep Workshop
Kim Felix, RHIA, CCS
AHIMA Approved ICD-10 Trainer
37
IOD Incorporated
May 2013
Circulatory System
• Question: What is the correct code assignment for pulmonary
hypertension? The default for pulmonary hypertension is code
416.8, Other chronic pulmonary heart diseases, but the inclusion
term for 416.8 is “pulmonary hypertension, secondary.” Code
416.0, Primary pulmonary hypertension, seems to be appropriate
for an unspecified pulmonary hypertension since “pulmonary
hypertension” is listed as an inclusion term.
• Answer: Assign code 416.8, Other chronic pulmonary heart
diseases, for pulmonary hypertension that is not documented
as primary or secondary. Since secondary pulmonary
hypertension is the most common type of pulmonary hypertension,
it is the default. Code 416.0, Primary pulmonary hypertension, is
only
y assigned
g
when the p
provider has specifically
p
y diagnosed
g
primary pulmonary hypertension.
Coding Clinic, 2nd quarter 2010, p. 10-11
75
Circulatory System
• CVA = Stroke = Cerebral Infarction
• All default to 434.91,, cerebral arteryy
occlusion, unspecified, with infarction
• Do NOT use code 436, Acute, but illdefined, cerebrovascular disease, when
the documentation states stroke or CVA
76
CCS Prep Workshop
Kim Felix, RHIA, CCS
AHIMA Approved ICD-10 Trainer
38
IOD Incorporated
May 2013
Circulatory System
• Question: According to Coding Clinic, Second Quarter 1989,
page 8, hospitals are not to report hemiplegia as an additional
diagnosis for patients who present with acute CVA if the
hemiplegia resolves prior to hospital discharge. Therefore,
hemiplegia is not being reported even though these patients
receive physical therapy or other treatment, which would
ordinarily signify reporting the hemiplegia based on the General
Rule for Reporting additional diagnoses. Could consideration be
given to allow coding this clinically significant diagnosis?
• Answer: Hemiplegia is not inherent to an acute
cerebrovascular accident (CVA). Therefore, it should be coded
even if the hemiplegia resolves, with or without treatment. The
hemiplegia
p g affects the care that the p
patient receives. Report
p any
y
neurological deficits caused by a CVA even when they have
been resolved at the time of discharge from the hospital. This
current advice supersedes information previously published in
Coding Clinic.
Coding Clinic, First Quarter 2010, p. 5
77
Circulatory System
• Aborted Stroke
– Patient has still suffered a stroke and an
acute stroke (434.91) should be coded,
even if tPA was given
– Selection of the correct code assignment
for a condition described as averted or
p
on whether the
aborted depends
condition actually occurred.
78
CCS Prep Workshop
Kim Felix, RHIA, CCS
AHIMA Approved ICD-10 Trainer
39
IOD Incorporated
May 2013
Circulatory System
• Late effects of CVA
– Neurological deficits that persist after the initial
onset of the stroke.
– May need a second code to describe the
remaining deficit if not clear in 438.xx code
• Patient with seizures s/p stroke 3 years ago.
– Use code 438.89 (other late effect) and 780.39
(seizures)
• Old CVA with no residuals
– Code v12.54
79
Circulatory System
• Myocardial Infarction
– Fifth digits
• 1- the FIRST episode of care, regardless of the
number of times a patient is transferred to and
from acute care facilities and occurring within the
8 week time frame
• 2- the subsequent episode of care, following the
initial episode. The patient has been discharged
from acute care (to home or long term care
facility) and returns for further observation,
evaluation or treatment for an MI and is still within
the
h 8 week
k time
i
fframe
• If the patient is readmitted more than 8 weeks after
the onset of the MI, do not use code 410.xx
80
CCS Prep Workshop
Kim Felix, RHIA, CCS
AHIMA Approved ICD-10 Trainer
40
IOD Incorporated
May 2013
Circulatory System
• Question: A patient with an acute ST elevation
lateral wall myocardial infarction (STEMI) was
initially seen at Hospital A and was immediately
transferred to Hospital B for an emergency
cardiac catheterization and percutaneous
transluminal coronary angioplasty (PTCA) with
stent insertion. What is the appropriate principal
diagnosis for Hospital B?
81
Circulatory System
• Answer: Assign code 410.51, Acute myocardial
infarction, Of other lateral wall, initial episode of
care, as the principal diagnosis. The acute
STEMI of the lateral wall had not resolved and
was still being treated at Hospital B. Assign code
414.01, Coronary atherosclerosis, of native
coronary artery, as a secondary diagnosis. This
advice is consistent with that previously
published in Coding Clinic Fifth Issue 1993,
page 14.
– Coding Clinic, Third Quarter 2009
82
CCS Prep Workshop
Kim Felix, RHIA, CCS
AHIMA Approved ICD-10 Trainer
41
IOD Incorporated
May 2013
Circulatory System
• Heart failure
– If congestive heart failure is documented
along with diastolic and/or systolic,
systolic 2 codes
must be used. There is no �combo’ code
• One code for CHF
• One code for diastolic and/or systolic heart failure
(determine if acute, chronic, acute on chronic or
unspecified)
83
Circulatory System
• Question: when a patient has a known history
of diastolic congestive heart failure (CHF), how
would this be coded?
• Answer: Assign code 428.32, diastolic heart
failure, chronic, and code 428.0, CHF
unspecified. This is coded as chronic due to the
patient’s know history.
– Coding Clinic 2008
84
CCS Prep Workshop
Kim Felix, RHIA, CCS
AHIMA Approved ICD-10 Trainer
42
IOD Incorporated
May 2013
Circulatory System
• Question: Is a diagnosis of systolic or diastolic
dysfunction coded the same as heart failure?
• Answer: No,, diastolic dysfunction
y
without
mention of heart failure is indexed to 429.9,
Heart disease, unspecified. It is not appropriate
to assume a patient is in heart failure when only
“diastolic dysfunction” or “systolic dysfunction” is
documented.
– Coding Clinic
Clinic, First Quarter 2009
85
Circulatory System
• Question: The patient is documented as having
systolic dysfunction with acute exacerbation of
congestive heart failure (CHF). Can this be coded
as acute
t systolic
t li heart
h t failure
f il
with
ith congestive
ti
heart failure?
• Answer: Assign code 428.0, Congestive heart
failure, unspecified and code 428.23, Systolic
heart failure, Acute on chronic. Acute exacerbation
of a chronic condition ((heart failure)) is coded as
acute on chronic.
– Coding Clinic First Quarter 2009
86
CCS Prep Workshop
Kim Felix, RHIA, CCS
AHIMA Approved ICD-10 Trainer
43
IOD Incorporated
May 2013
Circulatory System
• Question: A patient had an acute exacerbation of chronic
systolic congestive heart failure and severe mitral
regurgitation and aortic stenosis. How should this be
coded
coded?
• Answer: Assign codes 428.23, Acute on chronic systolic
heart failure; 428.0, Congestive heart failure, unspecified;
and 396.3, Mitral valve insufficiency and aortic valve
stenosis. As stated in Coding Clinic, Second Quarter 2000,
pages 16-17, “Do not make an assumption that the
congestive heart failure is rheumatic in nature.” Unless
ICD-9-CM directs the coder to assign the code for
“rheumatic” it is inappropriate to assign a code for rheumatic
congestive heart failure.
Coding Clinic, 1st qtr 2009, p. 18
87
Circulatory System
• Aborted/Impending MI
– If there is no myocardial injury/infarction
documented use 411
documented,
411.1
1 for an aborted or
impending MI. If there has been myocardial
injury/infarction, code the MI
• Selection of the correct code assignment for
a condition described as �averted’ or
�aborted’ depends
p
on whether the
condition actually occurred.
88
CCS Prep Workshop
Kim Felix, RHIA, CCS
AHIMA Approved ICD-10 Trainer
44
IOD Incorporated
May 2013
Circulatory System
• Chronic total occlusion of coronary artery 414.2
– Complete blockage of a coronary artery that has
been present for an extended period of time
(months years)
(months,
years).
– May be treated with angioplasty or stent placement
(usually drug-eluting)
– When total chronic coronary occlusion is present
with coronary atherosclerosis, assign code 414.2 as
an additional code. Do not assign 414.2 for an
acute coronary occlusion
• Coding Clinic, 2007
89
Circulatory System
• Endovascular Bioactive Coil
• Two new codes have been created to identify
endovascular embolization or occlusion of head or
neck vessels using bare metal coils (39
(39.75)
75) and
bioactive coils (39.76). Prior to this change, these
procedures were indexed to code 39.72,
Endovascular repair or occlusion of head and neck
vessels.
• Currently, there are two classes of coils: bare
platinum coils (BPCs) and bioactive coils.
90
CCS Prep Workshop
Kim Felix, RHIA, CCS
AHIMA Approved ICD-10 Trainer
45
IOD Incorporated
May 2013
Circulatory System
• New code 39.75 Endovascular embolization or occlusion of
vessel(s) of head or neck using bare coils
Bare metal coils
Bare p
platinum coils [[BPC]]
That for treatment of aneurysm, arteriovenous
malformation[AVM] or fistula performed.
• New code 39.76 Endovascular embolization or occlusion of
vessel(s) of head or neck using bioactive coils
Biodegradable inner luminal polymer coils
Coil embolization or occlusion utilizing
g bioactive coils
Coils containing polyglycolic acid [PGA]
That for treatment of aneurysm, arteriovenous
malformation [AVM] or fistula
91
Circulatory System
• Question: A 64-year-old patient with a right internal
carotid artery aneurysm underwent endovascular
embolization with six bioactive coils in the standard
as o until
u t complete
co p ete occ
occlusion
us o o
of tthe
ea
aneurysm
eu ys was
as
fashion
obtained. What is the appropriate code for the coil
embolization?
Answer: Assign code 39.76, Endovascular embolization
or occlusion of vessels(s) of head or neck using
bioactive coils, for the procedure
Coding Clinic, Fourth Quarter 2009
92
CCS Prep Workshop
Kim Felix, RHIA, CCS
AHIMA Approved ICD-10 Trainer
46
IOD Incorporated
May 2013
Circulatory System
• Don’t forget procedure codes 00.4x for specific
vascular procedures
• To be used in conjunction with other therapeutic
procedures
d
tto provide
id additional
dditi
l iinformation
f
ti (00
(00.6161
00.62, 00.66, 39.50, 38.10-38.18, 00.55, 00.63-00.65,
36.06-36.07, 39.90, 3609)
– 00.40-00.44 number of vessels treated or vessel
bifurcation
– 00.45-00.48 number of stents inserted
93
Circulatory System
• Question: this patient presented with femoral artery
aneurysm and underwent repair of the femoral artery
aneurysm with Dacron graft. During the surgery the femoral
artery was opened and a a thrombus was evacuated. ICD-9C provides unique codes ffor femoral
CM
f
artery aneurysm and
femoral artery thrombus. Would the thrombus and aneurysm
be reported separately?
• Answer: Assign code 442.3, other aneurysm of Lower
extremity for the femoral artery aneurysm. Do not assign an
additional code for the thrombus found during surgery. A
thrombus found and evacuated during aneurysm repair is
inherent in the diagnosis of aneurysm
aneurysm. Assign code 38
38.48,
48
resection of vessel with replacement, lower limb artery, for
the repair of the femoral artery aneurysm with Dacron graft
replacement
– Coding Clinic 2008
94
CCS Prep Workshop
Kim Felix, RHIA, CCS
AHIMA Approved ICD-10 Trainer
47
IOD Incorporated
May 2013
Circulatory System
• Question: Given the codes for chronic venous
embolism and thrombosis, when does DVT
become chronic?
• Answer: There are no specific timelines for
when DVT or any other condition becomes
chronic. The assignment of chronic DVT should
be based on provider documentation.
Coding Clinic, First Quarter, 2011
95
Respiratory System
• COPD
– Include obstructive chronic bronchitis (491.2x)
and emphysema (492.x)
(492 x)
– Unspecified COPD = 496
• Asthma
– Included in category 493.xx
96
CCS Prep Workshop
Kim Felix, RHIA, CCS
AHIMA Approved ICD-10 Trainer
48
IOD Incorporated
May 2013
Respiratory System
• Acute exacerbation of asthma versus status
asthmaticus
– Acute exacerbation = increased severityy of
asthma such as wheezing and SOB
– Status asthmaticus (intractable, refractory)=
failure to respond to therapy during an
asthmatic episode and is life threatening.
Status asthmaticus supercedes any type of
COPD and should be sequenced first if both
conditions are present.
97
Respiratory System
• Acute bronchitis with COPD
– Code 491.22 (Obstructive chronic bronchitis with
acute bronchitis). Do NOT code 466.0
• Acute bronchitis with COPD with acute
exacerbation
– Code 491.22 (Acute bronchitis supercedes the acute
exacerbation)
• Acute exacerbation of COPD
– Code 491.21 only
98
CCS Prep Workshop
Kim Felix, RHIA, CCS
AHIMA Approved ICD-10 Trainer
49
IOD Incorporated
May 2013
Respiratory System
• Acute Respiratory Failure
– Code 518.81 can be used as principal
diagnosis when it meets the definition of
principal diagnosis. This is a significant
change for some of the �old-time coders’.
• However, chapter-specific coding guidelines take
precedence (such as Obstetrics, Poisoning, HIV,
Newborn)
99
Respiratory System
Question: Is it appropriate to assign a code for
hypoxemia as an additional diagnosis when it is
associated with acute respiratory failure?
Answer: Do NOT assign hypoxemia as an
additional diagnosis when it is present or
associated with acute respiratory failure.
Hypoxemia would be considered inherent in
acute respiratory failure.
• Coding Clinic, 2006
100
CCS Prep Workshop
Kim Felix, RHIA, CCS
AHIMA Approved ICD-10 Trainer
50
IOD Incorporated
May 2013
Respiratory System
• Question: When a patient is admitted with an acute exacerbation of
chronic obstructive pulmonary disease (COPD) and an infection such
as pneumonia, is pneumonia always sequenced as the principal
diagnosis?
• Answer: Sequence either code 486, Pneumonia, organism
unspecified, or code 491.21, Obstructive chronic bronchitis, with
(acute) exacerbation, as the principal diagnosis, when the patient
is admitted with both conditions. The pneumonia and COPD are two
separate conditions that presented simultaneously. The pneumonia is
not the exacerbation of the COPD.
• The Official Guidelines for Coding and Reporting previously published
in Coding Clinic, Fourth Quarter 2008, page 303, states “In those rare
instances when two or more diagnoses equally meet the criteria for
principal diagnosis as determined by the circumstances of admission
admission,
diagnostic workup and/or therapy provided, and the Alphabetic Index,
Tabular List, or another coding guidelines does not provide
sequencing direction, any one of the diagnoses may be sequenced
first.”
Coding Clinic, First Quarter 2010, page 12-13
101
Respiratory System
• Pleural effusion
– Often is integral to an underlying condition
and is usually not coded
• EXCEPT when it is treated.
– i.e. Thoracentesis done for treatment of pleural
effusion
102
CCS Prep Workshop
Kim Felix, RHIA, CCS
AHIMA Approved ICD-10 Trainer
51
IOD Incorporated
May 2013
Respiratory System
• Effective October 1, 2009, code 488 has been revised and
expanded to create a new subcategory with two unique codes for:
– influenza due to identified avian influenza virus (488.0)
– influenza due to identified novel H1N1 influenza virus (488.1)
• These codes have been created to provide data capture for the
novel H1N1 influenza virus (swine flu virus), which was first
identified in April, after the March 2009 ICD-9-CM Coordination
and Maintenance Committee meeting.
• Similar to the guidelines for coding HIV infection, codes 488.0
and 488.1 should be assigned only for confirmed cases of
avian flu or H1N1 flu. Codes 488.0 and 488.1 are not assigned
when the final diagnostic statement indicates that the infection is
“
“suspected,”
t d ” “possible,”
“
ibl ” “lik
“likely,”
l ” or ““questionable.”
ti
bl ” Thi
This iis an
exception to the hospital inpatient guideline that directs the coder
to assign a code for a diagnosis qualified at the time of discharge
as suspected or possible as if it were established.
103
Respiratory System
• New code 488.0 Influenza due to identified avian
influenza virus
Avian influenza
d flu
u
Bird
Influenza A/H5N1
• New code 488.1 Influenza due to identified novel
H1N1 influenza virus
2009 H1N1 [swine] influenza virus
Novel 2009 influenza H1N1
Novel H1N1 influenza
N
Novel
l iinfluenza
fl
A/H1N1
Swine flu
104
CCS Prep Workshop
Kim Felix, RHIA, CCS
AHIMA Approved ICD-10 Trainer
52
IOD Incorporated
May 2013
Respiratory System
• Procedures
– Code 33.27, Closed [endoscopic] biopsy of lung, is
assigned when the biopsy is taken endoscopically
(through the bronchus) into the lung alveoli (i.e.
t
transbronchial
b
hi l bi
biopsy))
– Code 33.24, Closed [endoscopic] biopsy of
bronchus, is assigned when the biopsy is taken
endoscopically within the lumen of the trachea and
bronchus.
– Fiberoptic bronchoscopy with brushings and biopsies.
The pathology report describes bronchus and lung
tissue
tissue.
• Code 33.27 and 33.24 since both biopsies were performed.
– Coding Clinic, 2002
105
Respiratory System
• Procedures
– Question: The patient has complaint of fevers, cough and
an abnormal CT of the chest. The bronchoscope was
wedged
d d iin th
the lleft
ft llower llobe,
b superior
i segmentt and
da
bronchoalveolar lavage (BAL) was performed.
Transbronchial lung biopsies were performed in the left lower
lobe under fluoroscopic guidance where two biopsies were
taken. Endobronchial biopsies were taken at the superior
segment.
– Answer: Assign
g code 33.24,, Closed [endoscopic]
[
p ] biopsy
p y of
bronchus, for the BAL and endobronchial biopsies, and code
33.27, Closed endoscopic biopsy of lung, for the
transbronchial biopsies.
Coding Clinic, 3rd Qtr. 2011, p. 8
106
106
CCS Prep Workshop
Kim Felix, RHIA, CCS
AHIMA Approved ICD-10 Trainer
53
IOD Incorporated
May 2013
Respiratory System
• Mechanical Ventilation
• Question: Our question relates to patients admitted to a long term care
hospital on a T-piece or tracheostomy collar the day of the transfer, but
placed on mechanical ventilation that evening. How are the hours of
g counting
g hours at the
mechanical ventilation counted? Should we begin
start of the admission even though the patient is breathing through the
T-piece without mechanical ventilation, or are the hours counted from
the time the patient is on the vent?
• Answer: Yes, you should begin counting hours at the start of the
admission. All of the period of weaning is counted during the process of
withdrawing the patient from ventilatory support. The duration includes
the time the patient is on the ventilator, the weaning period and ends
when the mechanical ventilation is turned off (after the weaning period).
The fact that a T-piece is being used during the day does not affect code
assignment. A T-piece (trachcollar) trial involves the patient breathing
through a T-piece without ventilatory assistance for a set period of time.
Coding Clinic, 3rd Qtr 2010, p. 4
107
Sample Question
• A patient was discharged from the hospital with a
diagnosis of bronchial asthma. Upon reviewing the
record, the coder notes the patient was described as
having prolonged and intractable wheezing, airway
obstruction not relieved by bronchodilators, and a
decreased PAO2 lab value. The physician should be
queried to determine whether the code for
___________is appropriate as the principal diagnosis:
A. Acute viral Bronchitis
B. COPD
C. Respiratory failure
D. Status asthmaticus
108
CCS Prep Workshop
Kim Felix, RHIA, CCS
AHIMA Approved ICD-10 Trainer
54
IOD Incorporated
May 2013
Answer
• D. Status asthmaticus
109
Digestive System
• When minor adhesions are present and lysed
but do not cause symptoms or increase the
difficulty of the procedure, do not code the lysis
of adhesions
• When adhesions are strong, dense or create
problems or lengthen the operation, it is
appropriate to code the lysis of adhesions
110
CCS Prep Workshop
Kim Felix, RHIA, CCS
AHIMA Approved ICD-10 Trainer
55
IOD Incorporated
May 2013
Sample Question
• A patient was being treated for gastric ulcer with
hemorrhage, cirrhosis of liver, portal hypertension,
and esophageal varices. Of the following
medications,
di ti
which
hi h would
ld iindicate
di t a possible
ibl
complication or comorbid condition that would
impact DRG reimbursement?
A. BactrimВ®, 1 tablet q.i.d.
B. Darvocet-NВ®, 100 mg prn
C. HydroDIURILВ®, 50 mg PO daily
D. TagametВ®, 300 mg IM q 6 hrs
111
Answer
• A. Bactrim®, 1 tablet q.i.d.
112
CCS Prep Workshop
Kim Felix, RHIA, CCS
AHIMA Approved ICD-10 Trainer
56
IOD Incorporated
May 2013
Genitourinary System
• Urosepsis
– Defined as bacteria in the urine. Coded as 599.0
– Confusion as to whether this represents a true UTI
or septicemia from
f
a urinary source
– A diagnosis of septicemia can neither be assumed
nor ruled out on the basis of lab values alone
(positive blood culture)
– Signs/symptoms of septicemia where a query could
be necessary:
• Fever (or hypothermia),
hypothermia) malaise,
malaise fatigue,
fatigue
hyperventilation, tachypnea, tachycardia, change in
mental status, hypotension, metabolic acidosis, high
white cell count
113
Genitourinary System
• Question: What is the correct code assignment for a
urinary infection due to a suprapubic catheter? Would it be
classified differently than a urinary infection due to an
indwelling urethral catheter? There are no includes notes or
subterms in ICD
ICD-9-CM
9 CM that direct the coder to the proper
code assignment.
• Answer: Assign code 996.64, Infection and inflammatory
reaction due to indwelling urinary catheter along with code
599.0, Urinary tract infection, site not specified, for a UTI
due to suprapubic catheter infection. A suprapubic catheter
is an indwelling catheter that is placed directly into the
bladder through
g the abdomen. The catheter is normally
y
placed by an urologist and is inserted above the pubic bone
and attached to a drainage bag.
• Coding Clinic, Third Quarter 2009
114
CCS Prep Workshop
Kim Felix, RHIA, CCS
AHIMA Approved ICD-10 Trainer
57
IOD Incorporated
May 2013
Genitourinary System
• CKD
– Chronic kidney disease
• Stages II-IV,
IV ESRD (585
(585.x)
x)
• Renal Insufficiency
– Acute = 593.9
– Chronic = 585.9
– Unspecified
p
= 593.9
115
Genitourinary System
• Question: Clinical laboratory findings for
sodium and creatinine were elevated. After
workup,
p the MD diagnosed
g
acute kidney
y injury
j y
(AKI) secondary to volume depletion. How
should a non-traumatic AKI be coded?
• Answer: assign code 584.9, acute renal failure,
for a nontraumatic acute kidney injury (AKI).
– Coding Clinic 2008
116
CCS Prep Workshop
Kim Felix, RHIA, CCS
AHIMA Approved ICD-10 Trainer
58
IOD Incorporated
May 2013
Genitourinary System
• BEWARE:
– AKI = acute kidney injury
• 584.9
– AKI = acute kidney insufficiency
• 593.9
117
Genitourinary System
• Uterine Artery Embolization (UAE)
• Question: Please provide clarification on the correct code
assignment for a percutaneous uterine artery embolization. We find
the existing Index and Tabular instructions confusing. The recent
advice published in Coding Clinic,
Clinic Second Quarter 2009
2009, page 7
7, for
transcatheter embolizations seemed to be limited to coil
embolizations. We’re unsure whether a percutaneous transcatheter
uterine embolization using embospheres should be coded to 99.29,
Injection or infusion of other therapeutic or prophylactic substance; or
code 39.79, Other endovascular repair (of aneurysm) of other
vessels.
• Answer: Assign code 99.29, Injection or infusion of other
therapeutic or prophylactic substance, for uterine artery
embolization. The advice published in Coding
C
C
Clinic, S
Second
Quarter 2009 was specific to transcatheter coil embolizations.
Coding Clinic, First Quarter 2010, p. 21-22
118
CCS Prep Workshop
Kim Felix, RHIA, CCS
AHIMA Approved ICD-10 Trainer
59
IOD Incorporated
May 2013
Sample Question
• A patient undergoing hemodialysis for renal
disease in the outpatient unit of a hospital
develops what is believed to be heartburn. After a
few hours of observation,, he is admitted to the
hospital for further care. The consulting
cardiologist diagnoses this patient’s condition as
unstable angina. What is the principal diagnosis
for the hospital stay?
A. Complication of dialysis
B. Heartburn
C. Renal disease
D. Unstable angina
119
Answer
•
D. Unstable angina
120
CCS Prep Workshop
Kim Felix, RHIA, CCS
AHIMA Approved ICD-10 Trainer
60
IOD Incorporated
May 2013
Obstetrics
• 650- What is considered �normal’?
– Full term pregnancy
– Vaginal
g
delivery
y w/cephalic presentation, no fetal
rotation. No use of instruments
– Single, healthy infant
– No antepartum, delivery or postpartum complications
– Artificial rupture of membranes (73.09), other
manually assisted delivery (73.59), episiotomy (73.6),
other fetal monitoring
g ((75.34),
) injection
j
into spinal
canal, anesthetic (03.91), sterilization (66.21-66.29,
66.31-66.39)
121
Obstetrics
• With every delivery, the mom’s chart must
have a V27.x code. Not to be used on
subsequent visits or on newborn record
• 655.xx category (fetal abnormality
affecting the management of the mother)
– Assigned only when the fetal condition is
modifying the management of the mother
122
CCS Prep Workshop
Kim Felix, RHIA, CCS
AHIMA Approved ICD-10 Trainer
61
IOD Incorporated
May 2013
Obstetrics
• HIV Infection in Pregnancy
– Use 647.6x as principal dx if patient is
admitted with HIV-related illness followed by
042 and the code for the HIV-related illness
– Patients with asymptomatic HIV, admitted
during pregnancy should use 647.6x as
principal dx and V08 as a secondary dx
123
Obstetrics
• New codes as of October 1, 2008 for Maternal
and fetal complications due to In Utero
Procedures
• To be used on a mother’s
mother s record when the
management of a current pregnancy is affected
due to a complication of in utero surgery
performed during the current pregnancy.
– 679.0x- Maternal complications from in utero
procedure
– 679.1x- fetal complications from in utero procedure
124
CCS Prep Workshop
Kim Felix, RHIA, CCS
AHIMA Approved ICD-10 Trainer
62
IOD Incorporated
May 2013
Obstetrics
• Question: the patient had a normal spontaneous vaginal
delivery at 40 weeks gestation. The patient had an
episiotomy, which extended to a second-degree perineal
laceration The laceration was repaired with sutures
laceration.
sutures. How
should this be coded?
• Answer: Assign code 664.11, second-degree perineal
laceration as the principal diagnosis. Code v27.0 for the
outcome. Assign code 73.6, episiotomy and 75.69, repair
of OB laceration for the procedure performed. When an
episiotomy extends spontaneously to become a perineal
l
laceration,
ti
assign
i code
d 73
73.6
6 ttogether
th with
ith code
d 75
75.69.
69
Both codes are needed to completely describe this situation
– Coding Clinic 2008
125
Abortion
• Less than 22 weeks OR less than 500 gms
– Patient expels part or all of products of conception
– Missed abortion
abortion- less than 22 weeks and fetus is
retained in the uterus
• AKA blighted ovum
• Completed medically or surgically by MD as soon
as diagnosis is certain
– Threatened abortion- less than 22 weeks with
bleeding, without expulsion of fetus, without dilation
of cervix, so pregnancy continues
126
CCS Prep Workshop
Kim Felix, RHIA, CCS
AHIMA Approved ICD-10 Trainer
63
IOD Incorporated
May 2013
Musculoskeletal & Connective Tissue
• Pathologic fractures
– May be described as �spontaneous’
– Code 733.1x first with the etiology of the fracture as
secondary (i
(i.e.
e osteoporosis)
• Aftercare
– Use aftercare codes (v54.xx) for encounters after the
patient has completed active treatment of the fracture
and is receiving routine care during healing/recovery
• Active treatment includes: surgical treatment, ER
encounter, evaluation and treatment by a new physician
removal,
• Aftercare treatment includes: cast change or removal
removal of external or internal fixation device,
medication adjustment, and follow-up visits following
fracture treatment
127
Pathologic Fracture
•
•
•
Question: The patient is a 46-year-old man with multiple myeloma who was
admitted to the hospital for management of pathologic fracture of the proximal
shaft of the humerus. He also has impending pathologic fractures in multiple
places of his left femur, which are very painful. CT scan of the femur
demonstrated significant thinning of the medial wall, multiple cortical permeative
changes involving the femur neck and permeative changes of the ischium
ischium. The
patient is unable to bear weight; therefore the decision was made to undergo
prophylactic cephalomedullary trochanteric nail fixation of the left femur as well as
intramedullary nail fixation of the right humerus. How should this case be coded?
Answer: Assign code 733.11, Pathologic fracture of humerus, as the principal
diagnosis. Codes 203.00, Multiple myeloma, without mention of having achieved
remission, 731.8, Other bone involvement in disease classified elsewhere, and
V07.8, Other specified prophylactic measure, should be assigned as additional
diagnoses. It is not uncommon to perform prophylactic orthopedic procedures for
bone malignancies. However, it would be inappropriate to report code 733.15,
Pathologic fracture of other specified part of femur,
femur since it is not a confirmed
diagnosis. If an impending condition is not listed in the ICD-9-CM Alphabetic
Index, the condition described as impending or threatened is not coded.
For the procedures, assign codes 78.55, Internal fixation of bone without fracture
reduction, femur, and 78.52, Internal fixation of bone without fracture reduction,
humerus.
Coding Clinic, 2nd Quarter 2010, p. 6
128
CCS Prep Workshop
Kim Felix, RHIA, CCS
AHIMA Approved ICD-10 Trainer
64
IOD Incorporated
May 2013
Musculoskeletal & Connective
Tissue
• DJD
– 715.36, Osteoarthrosis, localized, not specified
whether primary or secondary, lower leg is assigned
for DJD of knee. When DJD affects only one site but
is not identified as primary or secondary, it is
assigned to 715.3x.
• Coding Clinic, 2003
• Note: Localized DJD includes bilateral
involvement of the same site
129
Musculoskeletal & Connective
Tissue
• Kyphoplasty w/vertebral biopsy
– Should
Sh ld a vertebral
t b l bi
biopsy b
be coded
d d separately
t l
when performed during a kyphoplasty?
– The biopsy is not an inherent part of the
procedure (kyphoplasty) and should be coded
separately if performed. Code 81.66, kyphoplasty
and 77.49,
77 49 biopsy of bone
bone, other
other.
• Coding Clinic, 2006
130
CCS Prep Workshop
Kim Felix, RHIA, CCS
AHIMA Approved ICD-10 Trainer
65
IOD Incorporated
May 2013
Musculoskeletal & Connective
Tissue
• Be careful coding
– Knee/Hip replacements
• Partial versus Total
– Knee/Hip revisions
• Revision of previous replacement or another total
replacement being done?
p
( ) of the jjoint is/are being
g
• Which component(s)
revised?
• If known, code type of components used (metal,
ceramic, polythylene)
131
Musculoskeletal & Connective
Tissue
• Spinal Fusions
– Site? (cervical, thoracic, lumbar, sacral)
– Technique? (anterior, posterior or both)
– How many # of vertebrae fused?
– Bone graft?
– Interbody spinal fusion device?
132
CCS Prep Workshop
Kim Felix, RHIA, CCS
AHIMA Approved ICD-10 Trainer
66
IOD Incorporated
May 2013
Sample Question
•
The following is an operative report to be coded:
Preoperative Diagnosis: Nonossifying fibroma
Postoperative Diagnosis: Aneurysmal bone cyst
Procedure: Excisional bone biopsy
The patient was brought to the operating room
room, and after adequate spinal
anesthesia, the right lower extremity was prepped and draped. A transverse
incision was carried down through the skin and subcutaneous tissue. The soft
tissues were dissected. The lesion was curetted from the bone, revealing a
cavity approximately 5.0 cm in length and 2.5 cm in width. The cavity was
irrigated and the margins electrocauterized. A specimen that was sent for frozen
section was consistent with aneurysmal bone cyst. The subcutaneous tissue
was closed with 2-0 Vicryl, and the skin was closed with 4-0 nylon. A sterile
dressing was applied. The patient tolerated the procedure well. Which of the
following CPT procedures is to be coded?
A Arthrotomy of ankle
A.
ankle, with biopsy for removal of loose body
B. Bone biopsy of leg or ankle area, deep
C. Curettage of bone cyst or benign tumor, tibia or fibula
D. Resection of tumor, radical; tibia
133
Answer
•
C. Curettage of bone cyst or benign
tumor tibia or fibula
tumor,
134
CCS Prep Workshop
Kim Felix, RHIA, CCS
AHIMA Approved ICD-10 Trainer
67
IOD Incorporated
May 2013
Decubitus/Pressure Ulcer
• New subcategory (707.2x) for stage
– Stage I-IV and unstageable
– The diagnosis of “pressure/decubitus ulcer”
should be based on the provider’s
documentation. However, the associated
code assignment may be based on other
clinicians (nursing, PT).
• Site of pressure ulcer should also be
coded (707.00-707.09)
135
Congenital Anomalies
• Codes from this chapter can be used
throughout a patient’s life
– Not
N t just
j t as newborn/infant
b /i f t
– May be identified later in life
136
CCS Prep Workshop
Kim Felix, RHIA, CCS
AHIMA Approved ICD-10 Trainer
68
IOD Incorporated
May 2013
Newborn/Perinatal
• V30-V39 should always be sequenced as principal
diagnosis for all birth admissions and will only be used
once in a patient’s life.
– Not to be used on subsequent
q
visits or on the
mother’s record
• If the newborn is transferred to another institution,
V30-V39 is not used at the receiving hospital. Code
the reason for the transfer.
• For coding and reporting purposes, the perinatal
period is defined as BEFORE birth through the 28th
day
y of life.
– If a condition begins before the 28th day of life and
continues after that, it can still be coded as a
perinatal condition.
137
Newborn/Perinatal
• Code all clinically significant conditions:
–
–
–
–
–
–
Clinical evaluation
Therapeutic treatment
Diagnostic procedures
Extended LOS
Increased nursing care/monitoring
Implications for future health care needs (not used for
adult patients)
138
CCS Prep Workshop
Kim Felix, RHIA, CCS
AHIMA Approved ICD-10 Trainer
69
IOD Incorporated
May 2013
Newborn/Perinatal
• Perinatal Morbidity due to maternal causes
– Code 760-763 are assigned ONLY when the
f t is
fetus
i affected
ff t d by
b the
th maternal
t
l condition
diti
• The fact that the mother has a medical condition or
experiences a complication of pregnancy or
delivery does NOT justify the routine assignment of
codes 760-763
139
Newborn/Perinatal
• Newborn Sepsis
– 771.81 (Septicemia/sepsis of newborn)
should be assigned to describe sepsis
• Not necessary to assign a code from 995.9x
and/or 038.xx for a newborn record
• Add code from 041.xx as a secondary diagnosis
to identify the organism
140
CCS Prep Workshop
Kim Felix, RHIA, CCS
AHIMA Approved ICD-10 Trainer
70
IOD Incorporated
May 2013
Newborn/Perinatal
• Apparent Life Threatening Event in Infant (ALTE)
• According to the National Institutes of Health, ALTE is an episode that is
frightening to the observer and is characterized by some combination of
– Apnea (central or obstructive)
– Color change (cyanotic, pallid, erythematous or plethoric)
– Change in muscle tone (usually diminished), and
– Choking or gagging
• Previously used terminology such as near-miss sudden infant death
syndrome (SIDS) or aborted crib death is outdated.
• A variety of congenital or acquired conditions can cause ALTE episodes.
Some of these conditions included GERD, pertussis, lower respiratory
tract infection, seizure, and UTI. In approximately half the ALTE cases, no
discrete cause can be identified despite extensive workup. A unique code
is important in order to track the true incidence of ALTE.
141
Newborn/Perinatal
• New code• 799.82 Apparent life threatening event
in infant
f
• Since the term ALTE describes a clinical syndrome,
additional code(s) should be reported for associated signs
and symptoms as noted in the Tabular instructions. This
code is intended for use only when no underlying diagnosis
has been identified.
– Coding Clinic, fourth quarter 2009
142
CCS Prep Workshop
Kim Felix, RHIA, CCS
AHIMA Approved ICD-10 Trainer
71
IOD Incorporated
May 2013
Newborn/Perinatal
• Question: a baby was delivered at 39.2 weeks with vacuum
extraction. The delivery record indicates bag/mask ventilation
and oxygen were provided for 1 minute. Should a procedure
code for the bag ventilation and oxygen be reported?
• Answer:
A
Th use off supplemental
The
l
t l oxygen and
d bag
b ventilation
til ti
is an integral part of the care of a newborn. Do not assign
additional codes for the brief use of bag/mask ventilation and
oxygen. Some newborns may experience difficulty
transitioning to extrauterine life and may require a period of
supplemental oxygen, and spontaneous respirations can often
be stimulated using a manual resuscitation bag and face-mask
(BMV). In most cases, infants respond to these measures with
no further interventions.
interventions In cases where they do not respond
and mechanical ventilation is required, the mechanical vent
codes should be used.
– Coding Clinic 2008
143
Signs and Symptoms
• When to use signs/symptom codes:
– No more specific diagnosis can be made
– Sign/symptom existed at time of initial encounter that was transient and
cause not determined (fever, resolved after one day)
– Provisional diagnosis was made in patient who failed to return for further
care
– Case was referred elsewhere for treatment before diagnosis was made
(transfers) (chest pain, transferred to Hosp A for cardiac cath)
– A more precise diagnosis was not available
– Sign/symptom that is present that represent important problems in medical
care, not routinely associated with a disease process (seizures)
– When a symptom is followed by a comparing/contrasting diagnosis, the
symptom code is sequenced first as principal diagnosis(i.e. abd pain due to
either GE or pancreatitis)
– When a sign/symptom is NOT integral to the disease process
– When its presence is significant in relationship to the condition or care given
(cirrhosis with ascites. Paracentesis is done for ascites)
144
CCS Prep Workshop
Kim Felix, RHIA, CCS
AHIMA Approved ICD-10 Trainer
72
IOD Incorporated
May 2013
Signs and Symptoms
• Patient admitted to the hospital with chest
pain. In the final diagnostic statement, the
provider documented chest pain
pain, most
likely of GI origin.
– Query the MD regarding the specific GI
condition. If the MD cannot provide a
definitive answer regarding the nature of the
GI condition, assign code 786.50 (chest pain,
unspecified)
ifi d) as the
th principal
i i l di
diagnosis
i
• Coding Clinic, 2007
145
Signs and Symptoms
• Ascites
– Code 789.5 has been expanded to separately
identify
y malignant
g
ascites ((789.51)) and other
ascites (789.59)
– The term �malignant ascites’ had previously
been an inclusion term at code 197.6. Now
under code 789.51 (Malignant ascites), code
first malignancy, such as 183.0, Ovarian CA,
197 6 Peritoneal CA
197.6,
146
CCS Prep Workshop
Kim Felix, RHIA, CCS
AHIMA Approved ICD-10 Trainer
73
IOD Incorporated
May 2013
Injuries
• Assign separate codes for each injury
unless
l
a combination
bi ti code
d iis provided
id d
• Multiple injury/fracture codes should only
be assigned if there is no specific code or
documentation available
147
Injuries
• Fractures
– Open fracture (open wound at fracture site)
• Compound,
C
d iinfected,
f t d missile,
i il puncture
t
and
d
with foreign body
– Closed fracture
• Comminuted, depressed, elevated,
greenstick, spiral and simple
• If a fracture is not identified as �open’ or
�closed’, the code for a closed fracture is
used
148
CCS Prep Workshop
Kim Felix, RHIA, CCS
AHIMA Approved ICD-10 Trainer
74
IOD Incorporated
May 2013
Injuries
• Fracture treatment
– Reduction
– Internal fixation
• Screws, pins, rods, staples or plates
– External fixation
• Casts, splints
149
Injuries
• Open wounds are considered complicated
when there is:
– Delayed
D l
dh
healing
li
– Delayed treatment
– Infection
– Foreign body in wound
150
CCS Prep Workshop
Kim Felix, RHIA, CCS
AHIMA Approved ICD-10 Trainer
75
IOD Incorporated
May 2013
Burns
• Sequence first the code that reflects the highest
degree of burn when more than one burn is
present
• Burns of the same local site (3 digit category 940947) but different degrees should be coded to the
highest degree recorded in the record
• Non-healing burns are coded as acute burns
• Code 958.3 (post-traumatic wound infection) as an
additional code for any infected burn site
site.
151
Debridement
• Excisional debridement (86.22)
– “Surgical” removal or cutting away of tissue (often done
with a scalpel)
– NOT defined by the term �sharp’
sharp
– Can be done in OR, ER or at bedside
– Can be performed by nurse, therapist, physician assistant
or physician
• Nonexcisional debridement (86.28)
– Brushing, scrubbing, irrigating, flushing or washing of
tissue, minor scissors removal of loose fragments
• (i.e.
(i versajet,
j t whirlpool)
hi l
l)
– NONOPERATIVE in nature
– Minor �snipping’ of tissue/loose fragments with scissors
152
CCS Prep Workshop
Kim Felix, RHIA, CCS
AHIMA Approved ICD-10 Trainer
76
IOD Incorporated
May 2013
Debridement
• The use of a scalpel (or blade) does NOT
indicate �excisional’ debridement
• The
Th description
d
i ti off th
the procedure
d
performed must support the type of
debridement performed
• When multiple layers are debrided (skin,
muscle,, fascia,, bone),
), code ONLY the
deepest layer debrided
153
Debridement
• Question: The Arobella Qoustic Wound
Therapy System uses an ultrasonic assisted
curette to debride wounds. How should the use
of this device be coded?
• Answer: Assign code 86.28, Non-excisional
debridement of wound, infection, or burn. The
device is used for mechanical debridement. If,
however, the provider documents that excisional
debridement was also performed in addition to
the use of this device, then code the excisional
d b id
debridement
t separately.
t l
Coding Clinic, 2nd quarter 2010, p. 11-12
154
CCS Prep Workshop
Kim Felix, RHIA, CCS
AHIMA Approved ICD-10 Trainer
77
IOD Incorporated
May 2013
Debridement
• Question: The provider indicated that the patient was admitted for IV
antibiotic therapy, as well as incision and drainage of a chronic nonhealing
ulceration on the plantar aspect of the right foot with recent development
of cellulitis and abscess in the area. The patient underwent incision and
drainage (I&D) of the right foot ulceration, with insertion of vancomycin
antibiotic
tibi ti b
beads.
d Th
The surgeon noted
t d th
thatt th
the ulceration
l
ti on th
the plantarl t
lateral aspect of the heel had a sinus tract, which tracked laterally. The
surgeon made an incision utilizing a #15 blade through the skin layers,
down to the sinus tract to open up the area. The ulceration, sinus tract,
and all fibrotic/necrotic and infected looking tissues were sharply excised.
The surgeon noted that the excisional debridement encompassed the
plantar fascial layer of the heel, but did not extend into bone or
capsular tissue. The area was packed with vancomycin-impregnated
antibiotic beads. What are the appropriate procedure codes for this case?
• Answer: Assign code 83
83.39,
39 Excision of lesion of other soft tissue
tissue,
for the excisional debridement of the fascia. Code 99.21, Injection of
antibiotic, may be assigned as an additional code for the use of
antibioticimpregnated beads.
Coding Clinic, 3rd quarter 2010, p. 11
155
Debridement- Versajet
• Question: A patient with a stage III sacral pressure ulcer
undergoes a Versajet debridement which includes skin,
subcutaneous fat and muscle. Previous Coding Clinic advice
published on Versajet debridement assigned code 86.28,
Nonexcisional debridement of wound, infection, or burn, for the
Versajet debridement of skin and subcutaneous tissue. Are all
debridements performed with the Versajet considered
“nonexcisional”? What is the appropriate procedure code
assignment in this case?
• Answer: Assign code 86.28, Nonexcisional debridement of
wound,
d infection,
i f ti
or burn,
b
ffor the
th Versajet
V
j td
debridement.
b id
t Versajet
V
j t
debridement is always considered nonsurgical mechanical
debridement. It does not involve cutting away or excising
devitalized tissue.
– Coding Clinic, Third Quarter 2009
156
CCS Prep Workshop
Kim Felix, RHIA, CCS
AHIMA Approved ICD-10 Trainer
78
IOD Incorporated
May 2013
Adverse Effects, Poisonings, Toxic Effects
• Adverse effect
– Drug correctly prescribed and properly administered
– Code the reaction first (tachycardia) plus the E-code
• Poisoning
– Error in prescription, overdose of drug intentionally taken,
nonprescribed drug taken with correctly prescribed and properly
administered drug (wrong dose, wrong substance, wrong route of
administration)
– Code the poisoning code first followed by a code for the manifestation.
Add drug abuse or dependence if documented
• Toxic effect
– Harmful substance is ingested or comes in contact with a person
– Code the toxic effect first (980-989) followed by the code that identifies
the result/manifestation of the toxic effect and the E-code
157
Sample Question
• A patient came to the emergency department with
hypotension and tachycardia. Upon examination, the
patient’s condition was determined to be the result of a
tetanus
teta
us to
toxoid
o d vaccine
acc e ad
administered
ste ed four
ou hours
ou s ea
earlier.
e
Which of the following is the appropriate sequencing?
A. Hypotension; tachycardia; and accidental
poisoning E code, tetanus toxoid
B. Hypotension; tachycardia; and therapeutic use
E code, tetanus toxoid
C. Poisoning due to tetanus toxoid and
therapeutic use E code, tetanus toxoid
D. Unspecified adverse reaction and undetermined
cause E code, tetanus toxoid
158
CCS Prep Workshop
Kim Felix, RHIA, CCS
AHIMA Approved ICD-10 Trainer
79
IOD Incorporated
May 2013
Answer
• B. Hypotension; tachycardia; and
th
therapeutic
ti use E code,
d ttetanus
t
t
toxoid
id
159
Complications
• Transplant complications
– Only assigned if the complication affects the
function of the transplanted organ
• Complications due to implant/device
– If a patient is admitted with an infection,
malfunction, etc due to a device, code the
complication code as principal diagnosis
160
CCS Prep Workshop
Kim Felix, RHIA, CCS
AHIMA Approved ICD-10 Trainer
80
IOD Incorporated
May 2013
Complications
• Question: a septic patient with respiratory failure,
underwent multiple attempts to insert a peripheral IV
for fluid resuscitation and frequent blood draws.
After multiple failed attempts
attempts, the patient
subsequently had ultrasound guidance to help with
the placement of a central venous catheter. The
MD’s diagnosis was “poor IV access”. What is the
diagnosis code for “poor IV access”?
• Answer: “Poor IV access” is not a medical
diagnosis. Assign a code for the underlying
condition that is being treated (e
(e.g.
g sepsis
sepsis,
respiratory failure, etc.)
– Coding Clinic 2008
161
Sample Question
• What is the diagnosis code assignment for
contraction of the anterior capsule causing the
intraocular lens implant to be displaced following
extraction
t ti off a cataract?
t
t? Th
The physician
h i i uses a
laser to repair the torn capsule and reposition the
lens.
A. 996.69
B. 996.53
C. 998.82
998 82
D. 998.89
162
CCS Prep Workshop
Kim Felix, RHIA, CCS
AHIMA Approved ICD-10 Trainer
81
IOD Incorporated
May 2013
Answer
•
B. 996.53
163
Late Effects
• Residual condition that remains after the
acute phase of the illness/injury has
passed
d
• No time periods
• Code first the residual condition then the
cause of the late effect
164
CCS Prep Workshop
Kim Felix, RHIA, CCS
AHIMA Approved ICD-10 Trainer
82
IOD Incorporated
May 2013
V codes
Admission
Counseling
History of (personal
and family)
Resistance
Aftercare
Dependence
Maintenance
Screening
Att ti
Attention
to
t
Di l i
Dialysis
M l dj t
Maladjustment
t
St t Post
Status
P t
Boarder
Donor
Newborn
Supervision of
Care (of)
Encounter for
Observation
Test (s)
Carrier of
Examination
Outcome of delivery
Therapy
Checking
Exposure
Pregnancy
Transplant(ed)
Contact
Fitting (of)
Problem
Unavailablity of
medical facilities
Contraception
Follow-up
Prophylactic
Vaccination
Convalescence
Health
Replacement
165
V code categories
•
•
•
•
•
•
•
•
Contact/exposure
– Exposure to TB V01.1
Inoculations/vaccinations
– Flu shot V03.81
Status
– S/P CABG V45.81
History (of)
– History of lung cancer V10.11
Screening
– Mammography screening V76.12
Observation
– Observation after MVA V71.4
Aftercare
– Colostomy takedown V55
V55.3
3
Follow-up
– Follow-up after surgery V67.09
•
•
•
•
•
•
•
Donor
– Kidney donor V59.4
Counseling
– Genetic counseling V26.33
Obstetrics and related conditions
– History of pre-term labor V23.41
Newborn, infant/child
– Normal newborn V30.00
Routine/administrative examinations
– Vision testing V20.2
Miscellaneous
– Prophylactic breast removal
V50.41
Nonspecific (limited use in outpatient
setting)
– History of arthritis V13.4
166
CCS Prep Workshop
Kim Felix, RHIA, CCS
AHIMA Approved ICD-10 Trainer
83
IOD Incorporated
May 2013
BMI
• BMI should only be reported as secondary
diagnoses
– For the BMI, code assignment may be based on
medical record documentation from clinicians
who are NOT the patient's provider (i.e. physician
or other qualified healthcare practitioner legally
accountable for establishing the patient's
diagnosis)
• This information is typically documented by other
clinicians involved in the care of the patient (e.g. a
dietician often documents the BMI
BMI.
• However, the associated diagnosis (such as
overweight or obesity) must be documented by
the patient's provider
167
Let’s Have Some Fun!
What’s wrong with these codes?
1. 174.4, V58.11, 197.0
2 357
2.
357.2,
2 250
250.60,
60 401
401.9
9
3. V27.0, 654,21, 644.21
4. 403.91, 585.6, 401.9, 562.10
5. 774.2, V30.00, V05.3
6. 250.70, 443.81, 250.62, 357.2
7 041
7.
041.11,
11 599
599.0,
0 401
401.9
9
8. 644.21, 650, v27.0
168
CCS Prep Workshop
Kim Felix, RHIA, CCS
AHIMA Approved ICD-10 Trainer
84
IOD Incorporated
May 2013
This is what she calls FUN?
1. 174.4, V58.11, 197.0- V58.11 must be principal dx
2. 357.2, 250.60, 401.9- 357.2 cannot be principal
3. V27.0, 654,21, 644.21- v27.0 cannot be principal
4. 403.91, 585.6, 401.9, 562.10- do not use 403.xx and 401.9 together
5. 774.2, V30.00, v05.3- V30.00 must be principal dx
6. 250.70, 443.81, 250.62, 357.2- inconsistent fifth digits on DM codes
7. 041.11, 599.0, 401.9- 041.11 cannot be used as principal dx
8. 644.21, 650, v27.0- cannot use 650 with complication code from
pregnancy chapter
169
Present on Admission (POA)
• Y = Yes (present at the time of inpatient
admission)
• N = No (not present at the time of inpatient
admission)
• U = Unknown (documentation is insufficient to
determine if condition is present on
admission
• W = Clinically undetermined (provider is unable
to clinically determine whether condition
was present on admission)
170
CCS Prep Workshop
Kim Felix, RHIA, CCS
AHIMA Approved ICD-10 Trainer
85
IOD Incorporated
May 2013
POA
•
Assign �Yes’ in the following examples:
• Condition present on admission: Patient admitted with lethargy,
fever, tachycardia, hypotension and altered mental status. Blood
culture grew out pseudomonas on day of admission. MD
d
documents
t �sepsis
�
i was presentt on admission
d i i as evidenced
id
db
by
the presenting signs and symptoms’.
• Condition diagnosed PRIOR to admission: Patient has a PMH of
angina and is treated with Imdur during the hospital stay.
• Condition diagnosed during the admission that was clearly
present before admission but not diagnosed until after the
admission occurred: Patient is admitted with cough and difficulty
breathing. After study, the final diagnosis is lung cancer.
• Condition confirmed after admission but documented as
suspected, possible, rule out, probable for a final diagnosis and
the diagnosis was suspected at the time of the inpatient
admission: Patient admitted with chest pain, rule out MI.
Diagnostic w/u could not confirm the MI. Final diagnosis is chest
pain, possible MI.
171
POA
Assign �Yes’ in the following examples:
• Final diagnosis is documented as impending or threatened and the
diagnosis is based on the symptoms or clinical findings that were
present on admission: Patient is admitted with chest pain, rule out
MI Final diagnosis is documented as impending MI
MI.
MI.
• Condition that develops during an outpatient encounter prior to a
written order for inpatient admission: Patient undergoes outpatient
surgery. During recovery, patient develops A fib and patient is
admitted as inpatient.
• Any acute condition present at time of admission: Patient admitted
from the ER. MD documents respiratory failure, pneumonia, ARF
and dehydration.
• Any
A chronic
h i condition
diti even th
though
h th
the condition
diti may nott be
b
diagnosed until after admission: ECHO is performed during
hospital stay and shows mitral regurgitation. MD confirms the
diagnosis of mitral regurgitation in the progress notes and
discharge summary.
•
172
CCS Prep Workshop
Kim Felix, RHIA, CCS
AHIMA Approved ICD-10 Trainer
86
IOD Incorporated
May 2013
POA
• Assign �Yes’ in the following examples:
• Combination code only identifies the chronic condition and not the
acute exacerbation: Patient admitted with decompensated CHF.
• Combination code where all parts of the combination code are
presentt on admission:
d i i
P ti t admitted
Patient
d itt d with
ith GI bl
bleed.
d EGD
demonstrates gastritis and MD documents GI bleed due to
gastritis.
• Comparative/contrasting diagnoses when both are present or
suspected at the time of admission: Abdominal pain documented
in the ER. MD documents abdominal pain may be due to either
pancreatitis or cholecystitis.
• Infection code that includes the causal organism when infection (or
signs
i
off iinfection)
f ti ) were presentt on admission:
d i i
P ti t in
Patient
i ER iis
diagnosed with pneumonia. Sputum culture taken in the ER
eventually grows out Klebsiella.
• Pregnancy complications/obstetrical conditions present on
admission: Patient admitted in labor and positive group B strep
culture was noted. Patient was treated with IV abx prior to
delivery.
173
POA
• Assign �Yes’ in the following examples:
• Newborn condition present at birth or developed in
utero including any condition that occurred during
delivery: Fetal bradycardia documented during
labor and is included in the final diagnosis on the
NB record.
• Any congenital condition/anomaly: Baby delivered
and Tetralogy of Fallot was noted by the MD
throughout the record.
• Any E code representing external cause of
injury/poisoning that occurred prior to admission:
Patient admitted with fracture of
femur due to fall down stairs.
174
CCS Prep Workshop
Kim Felix, RHIA, CCS
AHIMA Approved ICD-10 Trainer
87
IOD Incorporated
May 2013
POA
• Assign �No’ in the following examples:
• Any condition the provider explicitly documents as NOT present
at time of admission: Patient admitted with fever, elevated WBCs
and hypoxemia.
yp
Pneumonia is documented. Patient
progressively gets worse and on day 3 has hypotension,
tachycardia, pulse 132. Blood cultures grew out pseudomonas
on hospital day 5. MD documents �sepsis did not develop until
after admission’.
Pneumonia = Y, Sepsis = N
• Diagnosis contains the term possible, probable, suspected or rule
out for a final diagnosis and the diagnosis, symptoms or clinical
findings were NOT present on admission: On day 2, patient
develops chest pain
pain. MD documents chest pain due to possible
MI as final diagnosis.
• Final diagnoses is documented as impending or threatened and
the diagnosis is based on symptoms or clinical findings that were
NOT present on admission: On day 2, patient develops chest
pain. MD documents impending MI as final diagnosis.
175
POA
• Assign �No’ in the following examples:
• Acute conditions NOT present at the time of admission:
Patient develops V-tach on hospital day 3.
• Combination code where ALL parts of the combination
code were NOT present on admission: Patient with DM
developed OOC DM on hospital day 3.
Patient has gastric ulcer that does not start bleeding until
after admission.
• Pregnancy complications/OB conditions NOT present on
admission:
Patient delivers baby and has second degree laceration
176
CCS Prep Workshop
Kim Felix, RHIA, CCS
AHIMA Approved ICD-10 Trainer
88
IOD Incorporated
May 2013
POA
• Assign �No’ in the following examples:
• OB code including more than one diagnosis and all
parts of the diagnosis were NOT present on admission:
Patient with pre-existing
pre existing HTN is admitted and
developed pre-eclampsia after admission.
• Newborn condition that developed after
admission/delivery: NB developed diarrhea which was
believed to be due to hospital baby formula.
• Any E code representing an external cause of
injury/poisoning that occurred DURING the inpatient
stay: Patient experienced an adverse
reaction to med given on day 3.
177
POA
• Assign �U’ in the following example:
• *NOTE: �U’ should not be routinely used and only in very limited
circumstances.* When the documentation is unclear if the
condition was present on admission:
Patient admitted with fever and pneumonia. Patient rapidly
deteriorates and becomes septic. Discharge diagnosis lists sepsis
and pneumonia.
Documentation is unclear as to whether the sepsis was present on
admission or developed shortly after admission.
Pneumonia = Y, Sepsis = U
• Assign �W’ in the following example:
• When the documentation indicates that it cannot be clinically
determined whether or not the condition was present on admission:
Patient admitted in active labor. During the stay, a breast abscess is
noted when the mother attempts to breast feed. MD is unable to
determine whether the abscess was present on admission.
178
CCS Prep Workshop
Kim Felix, RHIA, CCS
AHIMA Approved ICD-10 Trainer
89
IOD Incorporated
May 2013
POA
• Question: a patient is admitted to the hospital
with a stage II pressure ulcer of the heel. During
the hospitalization, the pressure ulcer worsens
and becomes a stage III. Based on the new
Official Coding Guidelines, we would be
assigned the code for the highest stage for that
site. What would be the correct POA indicator
assignment for the stage III code?
• Answer: Assign �Y’ to the pressure ulcer stage
III code since this code is referring to a pressure
ulcer
l
th
thatt was presentt on admission
d i i rather
th th
than
a new ulcer.
– Coding Clinic 2008
179
POA
• Question: A patient is admitted with a subarachnoid hemorrhage
following an injury. At the time of admission there was no mention
of loss of consciousness. However, after admission the patient lost
consciousness for several hours. We assigned code 852.03,
Subarachnoid hemorrhage following injury without mention of open
intracranial wound, with moderate [1-24 hours] loss of
consciousness, as the principal diagnosis. What is the appropriate
POA indicator since the patient lost consciousness after
admission?
• Answer: Assign POA indicator “Y” since the injury occurred prior
to admission. Loss of consciousness is part of the natural history of
the disease process. In addition, the POA guideline governing
combination codes does not apply
pp y here, since this is not a
combination of diagnoses. The skull fracture (800- 804) and
intracranial injury (850-854) categories are unique, so this advice
only applies to these categories.
Coding Clinic, 2nd Quarter, 2010, p. 17
180
CCS Prep Workshop
Kim Felix, RHIA, CCS
AHIMA Approved ICD-10 Trainer
90
IOD Incorporated
May 2013
New Codes- Valid October 2012
• 041.4x E. coli expanded
• 173.xx basal cell, squamous cell
and unspecified malignant
neoplasms of skin (by site)
• 282.4x thalassemias
• 284.1x pancytopenias
• 365.7x mild, moderate, severe
and indeterminate stages of
glaucoma
• 415.13 saddle embolus of
pulmonary artery
• 425.1x cardiomyopathies
• 444.01 saddle embolus of
abdominal
bd i l aorta
t
• 512.8x various pneumothorax
• 516.3x various interstitial
pneumonitis
• 516.31 idiopathic pulmonary
fibrosis
• 518.51 acute respiratory failure
following trauma/surgery
pulmonary
y
• 518.52 other p
insufficiency, NEC, following
trauma/surgery
• 518.53 Acute and chronic
respiratory failure following
trauma/surgery
• 539.0x Complications of gastric
band surgery
• 539.8x Complications of bariatric
procedure
• 747.3x pulmonary artery
malformations
• 998.0x Post operative shock
181
New Codes- Valid October 2012
• 02.21 Insertion or replacement of
external ventricular drain
• 02.22 Intracranial ventricular shunt
or anastomosis
• 17.53-17.54
17 53 17 54 Percutaneous
atherectomy of extra- and intracranial vessels
• 17.55 Transluminal coronary
atherectomy
• 17.56 Atherectomy of noncornonary vessel(s)
• 35.05 Endovascular replacement
of aortic valve
• 35.06 Transapical replacement of
aortic valve
• 35.07 Endovascular replacement
of pulmonary valve
CCS Prep Workshop
Kim Felix, RHIA, CCS
AHIMA Approved ICD-10 Trainer
• 35.08 Transapical replacement of
pulmonary valve
• 35.09 Endovascular replacement of
unsp heart valve
• 39.77 Temporary therapeutic
endovascular occlusion of vessel
• 39.78 Endovascular implantation of
branching or fenestrated graft(s) in
aorta
• 43.82 Laparoscopic vertical
(sleeve) gastrectomy
• 68.24
68 24 Uterine artery embolization
[UAE] with coils
• 68.25 Uterine artery embolization
[UAE] without coils
91
IOD Incorporated
May 2013
Admission from Observation Unit
• When a patient is admitted to an observation unit for a
medical condition that requires an inpatient admission, the
principal diagnosis would be the medical condition which led
to this hospital admission
• When a patient is admitted from outpatient surgery due to a
complication, assign the complication code as principal dx
• If there is no complication or other condition after outpatient
surgery and the patient requires an inpatient admission,
assign the reason for the outpatient surgery as the principal
diagnosis
p
admission is another condition
• If the reason for the inpatient
unrelated to the surgery, assign the unrelated condition as
principal dx
183
183
Outpatient
p
Guidelines
184
CCS Prep Workshop
Kim Felix, RHIA, CCS
AHIMA Approved ICD-10 Trainer
92
IOD Incorporated
May 2013
Selection of First-listed Condition
• The term �first-listed’ condition is used in lieu of principal
dx
• Diagnosis are often not established at the time of the initial
encounter/visit May take 2 or more visits before the
encounter/visit.
diagnosis is confirmed.
• Outpatient surgery
– Code reason for the surgery as first-listed diagnosis, even if
the surgery is not performed due to contraindication
• Observation stay
– Assign
g code for the medical condition as first-listed diagnosis
g
– When patient develops complications after outpatient
surgery, requiring observation, code the reason for surgery
as the first-reported diagnosis followed by the
complications as secondary diagnoses
185
Uncertain Diagnosis
• DO NOT CODE diagnoses documented
as �probable, suspected, questionable,
rule
l outt or working
ki di
diagnosis’
i ’ or other
th
similar term. Rather, code the condition
to the highest degree of certainty, such as
signs, symptoms, or abnormal test
results.
186
CCS Prep Workshop
Kim Felix, RHIA, CCS
AHIMA Approved ICD-10 Trainer
93
IOD Incorporated
May 2013
Diagnostic Services
• For diagnostic tests that have been
interpreted by a physician, and the final
report is available at the time of coding
coding,
code any confirmed or definitive diagnoses
documented in the interpretation. Do not
code any related signs/symptoms as
additional diagnoses.
187
Preoperative Evaluations (only)
• Sequence first a code from category
v72.8,
72 8 other
th specified
ifi d examinations,
i ti
tto
describe pre-op consultations. Assign a
code for the condition to describe the
reason for surgery as an additional
diagnosis. Code also any findings related
to the pre-op evaluation
188
CCS Prep Workshop
Kim Felix, RHIA, CCS
AHIMA Approved ICD-10 Trainer
94
IOD Incorporated
May 2013
Ambulatory Surgery
• Code the diagnosis for which the surgery
is performed. If the postoperative dx
diff
differs
from
f
the
th preoperative
ti dx,
d select
l t th
the
postoperative dx for coding since it is the
most definitive.
189
Modifiers (Level I)
– Bilateral procedure was performed (-50)
• Caution: some codes describe bilateral procedures
• Typically NOT used for integumentary codes
– Reduced services (-52)
• No other code to accurately reflect the service provided
• Physician-directed reduction
– Distinct procedural service (-59)
• Used to report services not normally reported together
• Different session/encounter, different procedure, different
site (i
(i.e.
e removal of lesions from leg and biopsy of back
lesion.
– Excision of lesion code + biopsy code -59
190
190
CCS Prep Workshop
Kim Felix, RHIA, CCS
AHIMA Approved ICD-10 Trainer
95
IOD Incorporated
May 2013
Modifiers (Level I)
– Discontinued outpatient hospital procedure
PRIOR to administration of anesthesia (-73)
• Procedure stopped due to patient’s condition
• After surgical prep and sedation
• Prior to administration of anesthesia
– Discontinued outpatient hospital procedure
AFTER administration of anesthesia (-74)
• Procedure stopped due to patient’s condition
• After administration of anesthesia
• i.e. Intubation started or incision was made
191
Modifiers (Level II)
Anatomical Modifiers
–
–
–
–
–
–
–
–
LT left side
RT right
g side
E1 upper left, eyelid
E2 lower left, eyelid
E3 upper right, eyelid
E4 lower right, eyelid
FA left hand, thumb
F1 left hand
hand, second
digit
– F2 left hand, third digit
–
–
–
–
–
–
–
F5 right hand, thumb
F6 right hand, second digit
TA left foot, great toe
T1 left foot, second digit
T5 right foot, great toe
T6 right foot, second digit
LC left circumflex, coronary
artery
t
– RC right coronary artery
192
192
CCS Prep Workshop
Kim Felix, RHIA, CCS
AHIMA Approved ICD-10 Trainer
96
IOD Incorporated
May 2013
Surgical Package
(Global Surgery Payment)
• Surgical procedure
• Local, digital or topical anesthesia
• One related E/M encounter on date of, or
immediately prior to date of procedure (includes
H & P)
• Immediate postoperative care
• Preparation of orders
• Evaluation
E al ation of patient in postanesthesia recovery
reco er
area
• Typical postoperative follow-up care
193
Mutually Exclusive
• Applies to improbable or impossible
combination of codes
– Example:
E
l 69601
69601- revision
i i mastoidectomy,
t id t
resulting in complete mastoidectomy used
with code:
69604- Revision mastoidectomy; resulting in
tympanoplasty
194
CCS Prep Workshop
Kim Felix, RHIA, CCS
AHIMA Approved ICD-10 Trainer
97
IOD Incorporated
May 2013
E & M Coding
• Determine:
– Place of service (office, ER, nursing home)
– Type of service (consult, admission, NB)
– Patient status (new,
(new unless patient seen in last 3 years by MD
or MD or same specialty in same group)
– 3 key components
• History (problem focused, expanded, detailed, comprehensive)
• PE (problem focused, expanded, detailed, comprehensive)
• Medical decision making (straightforward, low, moderate, high)
– New patients require all 3 above components
– Established
E t bli h d patients
ti t usually
ll require
i 2 or 3 off above
b
components
– Some codes are based on time: critical care, prolonged
services
195
Integumentary Procedures
•
Types of repairs (12001-13160)
– Simple repair: used when the wound is superficial; eg, involving primarily
epidermis, dermis, and subcutaneous tissue and no deeper structures. The
wound closure involves closing one layer, and includes local anesthesia, and
chemical or electrocauterization of unclosed wounds
wounds.
– Intermediate repair: includes requires layered closure of one or more of the
deeper layers of subcutaneous tissue and superficial (nonmuscle) fascia.
Single-layer closure of heavily contaminated wounds, which required extensive
cleaning or removal of particulate matter also constitutes as intermediate
repair.
– Complex repair: requires more than layered closure, such as scar revision,
debridement (eg, traumatic lacerations or avulsions), extensive undermining,
stents or retention sutures
sutures. Necessary preparation includes creation of a defect
for repairs (eg, excision of a scar requiring a complex repair) or the
debridement of complicated lacerations or avulsions.
Steri-strips or bandages only are reported with the appropriate E/M code (no separate CPT
code)
196
CCS Prep Workshop
Kim Felix, RHIA, CCS
AHIMA Approved ICD-10 Trainer
98
IOD Incorporated
May 2013
Integumentary Procedures
• Lesions (10040-11646)
–
–
–
–
Benign or malignant?
What site/body
yp
part is involved?
Biopsy, excision, destruction, shaving?
How large is excised area? Not just lesion size
• Report each lesion excised separately
• Includes margins excised
• The size of the lesion is best found on the OPERATIVE
REPORT
• Excisional biopsy used when entire lesion is removed
– What type of closure was performed?
• Advancement flap codes include excision of lesions
197
197
Integumentary Procedures
• Integumentary Lesion Excisions
• The Integumentary System guidelines listed in
the CPT 2010 codebook define an excision as
the removal of a lesion, including margins,
through the full thickness of the dermis, and
including simple (nonlayered) closure and local
anesthesia.
• Code selection is determined by measuring the
greatest clinical diameter of the apparent
g
pp
lesion,
plus the margin required for complete excision,
prior to the procedure.
198
CCS Prep Workshop
Kim Felix, RHIA, CCS
AHIMA Approved ICD-10 Trainer
99
IOD Incorporated
May 2013
Integumentary Procedures
• Integumentary excision procedures may require simple, intermediate,
or complex closures. Simple repair is included in the lesion excision
and is not reported separately. However, repair by intermediate or
complex closures should be reported separately.
example if a malignant skin lesion on the left arm measuring 1
1.0
0
• For example,
cm is excised with 0.3cm margins (excised diameter 1.6 cm), and
requires a complex closure of the wound of 3.0cm length after
accounting for manipulation of the wound for closure, report CPT
code 11602, Excision, malignant lesion including margins, trunk,
arms, or legs; excised diameter 1.1 to 2.0 cm, for the excision; for
the repair, report code 13121, Repair, complex, scalp, arms, and/or
legs; 2.6 cm to 7.5 cm.
• Note that when the excision of benign or malignant skin lesions
(codes 11400-11446
11400 11446 and 11600-11646)
11600 11646) is performed in conjunction
with an adjacent tissue transfer (codes 14000-14302), only the
adjacent tissue transfer should be reported, as the excision is
included in this procedure.
199
Sample Question
• A patient has six actinic keratoses
destroyed cryosurgically. What should be
referenced under the CPT index?
A. Excision, lesion, skin (malignant)
B. Excision, lesion, skin (benign)
C. Lesion, skin, excision
D Lesion
D.
Lesion, skin,
skin destruction
200
CCS Prep Workshop
Kim Felix, RHIA, CCS
AHIMA Approved ICD-10 Trainer
100
IOD Incorporated
May 2013
Answer
•
D. Lesion, skin, destruction
201
Integumentary Procedures
• Debridement
– Percentage of body surface debrided
– Extent of skin debrided (full or partial
thickness)
– Depth of debridement (subcutaneous,
muscle, bone)
202
CCS Prep Workshop
Kim Felix, RHIA, CCS
AHIMA Approved ICD-10 Trainer
101
IOD Incorporated
May 2013
Integumentary Procedures
• Breast lesions
– Pre-op radiological markers (localization
wires)
i ) often
ft used
d prior
i tto biopsies
bi
i and
d
excisions
• Use codes 19290-19291 for placement
• Use codes 19125-19126 for excision of
breast lesion with pre-op marker previously
placed
203
Integumentary Procedures
• Skin replacement/substitutes
– Surgical preparation
• 15002-15005
15002 15005
• Does donor site require repair?
– Grafts (Split-thickness, full-thickness,
allograft/donor, autograft, skin substitute,
xenograft/nonhuman donor)
• 14000-14350
14000 14350
– Tissue-cultured Epidermal Autograft
• 15150-15157
204
CCS Prep Workshop
Kim Felix, RHIA, CCS
AHIMA Approved ICD-10 Trainer
102
IOD Incorporated
May 2013
Integumentary Procedures
• Example:
– 10 sq cm epidermal autograft to face
from back.
• 15115
205
Musculoskeletal System
• Whether reporting the excision or radical resection of soft tissue tumors
from the subcutaneous, fascial or subfascial layer, appreciable vessel
exploration and/or neuroplasty should be reported separately. Simple
and intermediate repair closures are included in the excision
procedures,, but if complex
p
p
repairs
p
with extensive undermining
g or other
techniques are performed to close a defect created by a lesion excision,
the complex repair codes are reported separately.
• The excision of musculoskeletal lesions (tumors), includes the dissection
or elevation of tissue planes in order to allow resection of the tumor, and
therefore, those services are not reported separately. The code
selection for musculoskeletal lesion excisions is determined by
measuring the greatest diameter of the tumor, in addition to the
narrowest margin required for the complete excision of the tumor, based
on the p
physician's
y
jjudgment,
g
at the time of the excision.
• The radical resection of soft tissue tumors may be confined to a specific
layer, for instance the subcutaneous or subfascial tissue, or it may
involve the removal of tissue from one or more layers. Radical resection
of soft tissue tumors is most commonly used for malignant or very
aggressive benign tumors.
206
CCS Prep Workshop
Kim Felix, RHIA, CCS
AHIMA Approved ICD-10 Trainer
103
IOD Incorporated
May 2013
Musculoskeletal System
• Musculoskeletal Lesion Excisions
• Musculoskeletal lesion excision codes pertain to
subcutaneous, superficial, or deep soft tissues under the
skin which may include subcutaneous fat
skin,
fat, fascia
fascia, muscle
and bone. Soft tissue excision codes are dispersed
throughout the CPT 2010 musculoskeletal section and are
categorized by anatomic site.
• When coding musculoskeletal procedures, it is important to
note that the excision must meet the criteria listed in the
code descriptor. For example, in order to report code
26116 Excision,
26116,
E i i
tumor,
t
soft
ft tissue,
ti
or vascular
l
malformation, of hand or finger; subfascial
(eg,intramuscular); less than 1.5 cm, the tumor must be
down to the muscle (ie, located between the fascia and
muscle) or be intramuscular, such as a muscle sarcoma.
207
Examples
• The following are the different types of excisions, as listed in the
guidelines:
• Subcutaneous soft tissue tumors: involve the simple or marginal
resection of tumors confined to subcutaneous fatty tissue below
the skin,
skin but above the deep fascia
fascia.
• Fascial or subfascial soft tissue tumors: involve the resection of
tumors confined to the tissue within or below the deep fascia, but
not involving the muscle or bone. Included are digital (ie, fingers
and toes) subfascial tumors that involve the tendons, tendon
sheaths, or joints of the digit.
• Radical resection of soft tissue tumors: involve the resection of
a tumor, usually malignant, with wide margins of normal tissue.
• Radical
R di l resection
ti
off bone
b
tumors:
t
i
involve
l th
the resection
ti off th
the
tumor with wide margins of normal tissue. Radical resection of
bone tumors is usually performed for malignant tumors or
very aggressive tumors.
(See CPT Assistant February 2010.)
208
CCS Prep Workshop
Kim Felix, RHIA, CCS
AHIMA Approved ICD-10 Trainer
104
IOD Incorporated
May 2013
Examples
• Subcutaneous soft tissue tumors: usually benign and
resected without removing a significant amount of
surrounding normal tissue.
• Fascial or subfascial soft tissue tumors: usually benign,
i
involve
l ffascia
i and/or
d/ muscle,
l and
d resected
t d without
ith t removing
i
a significant amount of surrounding normal tissue.
• Digital (ie, fingers and toes) subcutaneous tumors:
adjacent to but not breaching the tendon, tendon sheath, or
joint capsule.
• Digital (ie, fingers and toes) fascial or subfascial
tumors: involve the tendon, tendon sheath, or joint
capsule
capsule.
• Radical resection of soft tissue tumors: most commonly
used for malignant tumors, and extremely aggressive
benign tumors in which wide margins of normal tissue are
excised.
209
Examples
• Question: May I report code 19260, Excision of
chest wall tumor including ribs with Modifier 52
appended, if the excision of a 10cm chest wall
mass did not include removing the ribs?
• Answer: No. It would not be appropriate to report code
19260, Excision of chest wall tumor including ribs, with
Modifier 52 appended, as the procedure did not involve
removal of the ribs. If the procedure involves removing a
chest wall tumor without the ribs, it would be more
appropriate to report a musculoskeletal tumor excision
code,
d such
h as code
d 21557,
21557 Radical
R di l resection
ti off tumor
t
(eg, malignant neoplasm), soft tissue of neck or anterior
thorax; less than 5 cm, depending on the depth, size,
and malignant or benign nature of the lesion.
210
CCS Prep Workshop
Kim Felix, RHIA, CCS
AHIMA Approved ICD-10 Trainer
105
IOD Incorporated
May 2013
Examples
• Question: What would be the appropriate CPT code for excision of a
sebaceous cyst on the scalp or on the face that is subdermal or
deeper?
• Answer: Integumentary lesion excision codes pertain to the epidermis,
dermis and subcutaneous tissue
dermis,
tissue, while musculoskeletal lesion excision
codes pertain to subcutaneous, superficial or deep soft tissues. Code
ranges 11400-11446 and 11600-11646 represent lesions that normally
occur on the surface of the skin (epidermis) or near the surface of the skin
(dermis), compared to the type of lesion (or tumor) that occurs in the
subfascial or fascial tissue, muscles and joints, as listed in the
musculoskeletal section. A sebaceous cyst is a skin lesion and may be
very large, distending the skin and pushing into the subcutaneous fatty
tissue, but it is a skin lesion, and therefore, should be coded using the
g
y lesion excision codes, depending
p
g on the size of the cyst.
y
integumentary
• Code range 21011-21016 lists the excision codes for soft tissue tumors
subcutaneous and subfascial on the face or scalp. When coding
musculoskeletal procedures, it is important to note that the excision must
meet the criteria listed in the code descriptor. The physician must
determine and document the depth of the excision to determine whether
the integumentary system or musculoskeletal system CPT codes are
appropriate.
211
Sample Question
• The following CPT codes appear:
19120 Excision of cyst, fibroadenoma, or other benign or malignant
tumor, aberrant breast tissue, duct lesion, nipple, or areolar lesion
(except 19140), male or female, one or more lesions
19125 Excision of breast lesion identified by preoperative
placement of radiological marker; single lesion
19290 Preoperative placement of needle localization wire, breast
The patient underwent a needle localization with excision of a right
breast lesion. Pathology revealed diffuse fibrocystic disease. Which
of the following is the appropriate coding for this procedure?
A. 19125; 19290
B 19125
B.
C. 19120; 19125
D. 19120; 19290
212
CCS Prep Workshop
Kim Felix, RHIA, CCS
AHIMA Approved ICD-10 Trainer
106
IOD Incorporated
May 2013
Answer
•
A. 19125; 19290
– Notice there are no modifiers???
213
213
Fractures/Dislocations
• Treatment can be open, closed/percutaneous
• Closed treatment
– Fracture/dislocation site is not surgically opened
• Open treatment
– Surgically opened to visualize and allow treatment
• Skeletal fixation
– Neither open nor closed. Pins placed across fracture
using x-ray guidance
• Manipulation
M i l ti
– Reduction or restoration of a fracture/dislocation
214
CCS Prep Workshop
Kim Felix, RHIA, CCS
AHIMA Approved ICD-10 Trainer
107
IOD Incorporated
May 2013
Fractures/Dislocations
• Example:
– Patient arrives to ER with left intertrochanteric
fracture and left tib/fib fracture. ORIF was
performed on both fractures
• 27244 -LT, 27758 -LT
215
Bronchoscopy and Biopsy
•
•
•
•
Endobronchial Biopsy: 31625
Transbronchial Biopsy: 31628
Transbronchial needle aspiration biopsy: 31629
Catheter aspiration of tracheobronchial tree at
bedside: 31725
• NOTE: Cell washings/brushings are NOT
considered biopsies
216
CCS Prep Workshop
Kim Felix, RHIA, CCS
AHIMA Approved ICD-10 Trainer
108
IOD Incorporated
May 2013
Nasal Procedures
• Hemorrhage control (30901-30999)
– Packing, ligation, cauterization
– Anterior: was hemorrhage simple or complex?
• Inserts gauze packing or performs cauterization
– Posterior: was control of hemorrhage initial or
subsequent procedure?
• Typically a more severe bleed
• Inserts nasal stents,, tampons,
p
, balloon catheters
or patient taken to OR for ligation
217
Cardiac Pacemakers/ Defibrillators
(33202-33249)
• Questions to ask:
–
–
–
–
–
Permanentt or temporary?
P
t
?
Approach? (transverse or thoracotomy)
Type of device? (electrodes and/or generator)
Electrode placement? (atrial and/or ventricle)
Initial placement, revision, removal, or
replacement?
l
t?
– Skin pocket revision?
218
CCS Prep Workshop
Kim Felix, RHIA, CCS
AHIMA Approved ICD-10 Trainer
109
IOD Incorporated
May 2013
Venous Access Devices
• Insertion (36555-36571)
–
–
–
–
–
Tunneled (under skin), Hickman, Broviac, Groshong
Non tunneled , triple lumen, PICC
Centrally placed
placed, subclavian
subclavian, jugular
jugular, IVC
Peripheral
Age?
• Repair (36575, 36576)
– Without replacement
• Replacement (36578-36585)
– Partial versus complete
– With/without port or pump
• Removal (36589, 36590)
– Use for tunneled catheters only, non-tunneled cath removals are NOT
reported separately
• Guidance (77001-fluoro, 76937- ultrasound)
219
Digestive Procedures
• Hernias (49491-49611)
– Initial vs. recurrent
– Reducible vs. strangulated
– Use code 49568 for incisional and
ventral hernia repairs
220
CCS Prep Workshop
Kim Felix, RHIA, CCS
AHIMA Approved ICD-10 Trainer
110
IOD Incorporated
May 2013
Digestive Procedures
• Code biopsy if single lesion is biopsied (but not excised)
• Code biopsy once if multiple biopsies are done (from
same or different lesions) and none of the lesions are
excised
• Code only the excision if a biopsy of lesion is done and
the balance of the same lesion is then excised
• Code both the biopsy and excision if both are performed
and if the biopsy is taken from a lesion DIFFERENT from
that which is excised; AND if the code for the excision
does not include the phrase �with or without biopsy’. If
that is the case, then a separate biopsy code should not
be used
• Use Modifier -59 to explain when coding both biopsy and
removal of a different lesion
221
Genitourinary
• Different approaches include:
– Cystoscopy, urethroscopy,
cystourethroscopy ureteroscopy
cystourethroscopy,
ureteroscopy,
pyeloscopy, renal endoscopy
• BEWARE:
– Ureteral versus Urethral procedures!
222
CCS Prep Workshop
Kim Felix, RHIA, CCS
AHIMA Approved ICD-10 Trainer
111
IOD Incorporated
May 2013
Abortion
• Spontaneous abortion in any trimester and
completed surgically: 59812
• Missed abortion occurring in the first
f
trimester: 59820
• Induced abortion with curettage and
evacuation: 59851
• Induced abortion evacuation: 59841
• Abortion by D & C : 59840
– Nonobstetrical D & C : 58120
223
Spinal Procedures
• Epidural injections are
just that- injection into
the epidural space
space.
• Root injections differ in
that the approach is
more difficult. It is
through the nerve root
foramen versus
translaminer.
• In addition,
addition add
fluoroscopy code when
performed.
Approach
Site
CPT
Epidural
Injection
Cervical/
Thoracic
62310
Epidural
Injection
Lumbar/
Sacral
62311
Root
Injection
(Lt and Rt)
Cervical/ 64479-1st level
Thoracic
64480- each
(Lt and Rt) addt’l level
Root
Injection
(Lt and Rt)
Lumbar/ 64483- 1st level
Sacral
64484- each
(Lt and Rt) addt’l level
224
CCS Prep Workshop
Kim Felix, RHIA, CCS
AHIMA Approved ICD-10 Trainer
112
IOD Incorporated
May 2013
Eye Procedures
• Cataract extraction
– ICCE Intracapsular cataract extraction
– ECCE Extracapsular cataract extraction
– Anterior chamber intraocular lens (IOL)
– Posterior chamber intraocular lens (IOL)
• Do NOT code injection done in conjunction with
cataract surgery.
g y Considered p
part of the
procedure.
225
Eye Procedures
• Cataract removal includes codes: 6683066984
– DO NOT code
d the
th following
f ll i procedures
d
separately:
•
•
•
•
•
Anterior/Posterior capsulotomy
Enzymatic zonulysis
Iridectomy/iridotomy
Lateral canthotomy
Subconjunctival/sub-tenon injections
226
CCS Prep Workshop
Kim Felix, RHIA, CCS
AHIMA Approved ICD-10 Trainer
113
IOD Incorporated
May 2013
Infusions/Injections
• Injection = 15 minutes or less
• Infusion (one hour) = 31-90 minutes
– 30 minutes or less- do not report
– “Each
Each additional
additional” must be 31 minutes or greater
• Report only 1 initial infusion (even if multiple infusion,
injections or a combination of both are performed), unless 2
separate IV sites are used
– Choose the initial code that best describes the key reason
for the encounter versus the �order’ in which they occurred
– Therapy infusions take precedence over �hydration’
– Therefore, any hydration therapy administered subsequent
to the initial drug/substance infusion, only �each additional’
hydration code(s) should be used
227
Infusions/Injections
• Hydration infusion note
– Hydration codes are NOT reported when IV
hydration is running while the therapeutic
agent infusion.
– In other words, hydration codes can only be
sequential (pre- and post-) but NOT
concurrent
228
CCS Prep Workshop
Kim Felix, RHIA, CCS
AHIMA Approved ICD-10 Trainer
114
IOD Incorporated
May 2013
Infusions/Injections
• Example:
– For example, if intravenous hydration as
d
described
ib d b
by codes
d 96360 and
d 96361 iis given
i
from 11:00 pm to 2:00 am, code 96360 would
be reported once for the first hour and code
96361 would be reported twice (once for each
additional hour of hydration intravenous
infusion)
infusion).
229
Infusions/Injections
• Question: On the basis of the infusion guidelines for
facilities, in the following scenario, is IV hydration primary
to an IV push in the hierarchy? A patient receives an IV
hydration from 1:00 pm to 3:00 pm and two IV pushes of the
same drug
d
att 1:30
1 30 pm and
d3
3:30
30 pm.
• No. The guidelines indicate that "therapeutic, prophylactic,
and diagnostic services are primary to hydration services."
Therefore, the intravenous push administrations of the
same drug are primary to the hydration service.For facility
reporting pertaining to this specific scenario, the
intravenous push drug administration performed at 1:00
pm, code 96374 is considered the "initial" service and is
primary to the hydration service.
service Code 96376 is for the 1:30
pm IV push. The 3:30 pm IV-push administration of the
same drug is reported with code 96376.
230
CCS Prep Workshop
Kim Felix, RHIA, CCS
AHIMA Approved ICD-10 Trainer
115
IOD Incorporated
May 2013
Infusions/Injections
• More on the answer:
– The 2-hour duration of the intravenous "hydration"
infusion from 1:00 pm to 3:00 pm is provided as a
secondary service to a different initial service (code
96374). However, hydration may not be a concurrent
service. In this instance, some portion less than 16
minutes of the 2 hours of "hydration" infusion, was
concurrent. Therefore, the hydration is reported using
code 96361 once. This reporting instruction is outlined
i th
in
the parenthetical
th ti l note
t ffollowing
ll i code
d 96361
96361, where
h
users are directed to report add-on code 96361 in
conjunction with code 96374 and beneath code 96360,
whereby it notes that hydration is not a concurrent
service.
231
Infusions/Injections
• The Q&A was offered to clarify that while some portion of the IVpush administration may occur concurrent to the hydration
infusion, it is not appropriate to report code 96368. Instead, code
96361 should be reported. The instruction does not suggest or
infer that the time of the IV-push
IV push administration is to be deducted
from the total hydration infusion interval.
• To further clarify, for facility reporting specific to this scenario, the
intravenous-push drug administration performed at 1:30 pm, is
considered the initial service, ie, primary to the hydration service.
Therefore, code 96374, Therapeutic, prophylactic, or diagnostic
injection (specify substance or drug); intravenous push, single or
initial substance/drug, is reported for the 1:30 pm IV-push
administration of the same drug.
g Add-on code 96376, Therapeutic,
p
prophylactic, or diagnostic injection (specify substance or drug);
each additional sequential intravenous push of the same
substance/drug provided in a facility (List separately in addition to
code for primary procedure) is reported for the 3:30 pm push of
the same drug.
232
CCS Prep Workshop
Kim Felix, RHIA, CCS
AHIMA Approved ICD-10 Trainer
116
IOD Incorporated
May 2013
Infusions/Injections
• Furthermore, the 2-hour duration of the intravenous
hydration infusion from 1:00 pm to 3:00 pm is
provided as a secondary service to a different initial
service (96374). In this instance, the 2 hours of
hydration infusion is reported twice using code
96361 (2 units) as it is reported once for each hour
of the 2-hour infusion. This reporting instruction is
outlined in the parenthetical note following code
96361, in which users are directed to report add-on
code 96361 in conjunction with code 96374.
Nevertheless, hydration
y
may
y not be a concurrent
service and, therefore, the duration of the pushes
could affect the units reported for hydration. Note
that overlapping time may not be included.
• **CPT Assistant, May 2010, page 8
233
Cardiac Catheterization
• CPT Definition of cardiac catheterization:
– Introduction, positioning, repositioning of
catheter
– Recording of intracardiac/intravascular
pressure
– Obtaining blood samples for blood gases or
dilution curves
– Cardiac
C di output
t t measurements
t with
ith or without
ith t
electrode catheter placement
– Final evaluation and report of procedure
234
CCS Prep Workshop
Kim Felix, RHIA, CCS
AHIMA Approved ICD-10 Trainer
117
IOD Incorporated
May 2013
Cardiac Catheterization
•
•
•
•
•
93451 Right heart catheterization
93452 Left heart catheterization including injection(s) for LVgram, imaging S&I
93453 Combined right and left heart catheterization including intraprocedural injection(s) for
LVgram, imaging S&I
93454 Catheter placement in coronary artery(s) for coronary angiography, including
injection(s) for coronary angiography, imaging S&I
93455 with catheter placement(s) in bypass graft(s) (internal mammary, free arterial venou
grafts) including injection(s) for bypass graft angiography
93456 with right heart cath
93457 with catheter placement(s) in bypass graft(s) (internal mammary, free arterial,
venous grafts) including intraprocedural injection(s) for bypass graft angiography and
right heart catheterization
93458 with left heart cath including injection(s) for LVgram, when performed
93459 with left heart cath including injection(s) for LVgram, when performed, catheter
placement(s) in bypass graft(s) (internal mammary, free arterial, venous grafts) with
yp
g
graft angiography
g g p y
bypass
93460 with right and left heart cath including injection(s) for LVgram
93461 with right and left heart cath including intraprocedural iniection(s) for LVgram,
catheter placement(s) in bypass graft(s) (internal mammary, free arterial, venous grafts
with bypass graft angiography
93462 Left heart cath by transseptal puncture through intact septum or by transapical
puncture (List separately in addition to code for primary procedure)
235
Cardiac Catheterization
• Most common procedures:
– Right and left heart cath with coronary
angiography and LVgram 93460
– Left heart cath with coronary angiography
93458
– Left heart cath with coronary angiography and
LVgram 93458
– Left
L ft heart
h t cath
th with
ith LVgram
LV
93452
– Left heart cath 93452
– Right heart cath 93451
236
CCS Prep Workshop
Kim Felix, RHIA, CCS
AHIMA Approved ICD-10 Trainer
118
IOD Incorporated
May 2013
PTCA/Stent/Atherectomy
Procedure
Single
Vessel
Each
addt’l
vessel
**Assigned PER
VESSEL, even when
St t
92980 92891
multiple strictures are Stent
treated within the
same vessel
Coronary
92982 92984
angioplasty**
**Include when
performed with a
� tti b
�cutting
balloon’
ll
’
Coronary
atherectomy
92995 92996
237
Sample Question
• A cardiovascular procedure that is unfamiliar to the
coder is performed, and the procedural name used
by the physician does not appear in the CPT index.
In such a situation
situation, what should the coder do first?
A. Ask the physician to review the codes in the
cardiovascular section of CPT
B. Assign a similar cardiovascular procedure code
C. Postpone coding the specific procedure until a
code is established by the AMA
D. Use an unlisted procedure code from the
cardiovascular section
238
CCS Prep Workshop
Kim Felix, RHIA, CCS
AHIMA Approved ICD-10 Trainer
119
IOD Incorporated
May 2013
Answer
• A. Ask the physician to review the
codes
d iin th
the cardiovascular
di
l section
ti off
CPT
239
Other Tidbits of Info
240
CCS Prep Workshop
Kim Felix, RHIA, CCS
AHIMA Approved ICD-10 Trainer
120
IOD Incorporated
May 2013
Rule Out
• Rule out pneumonia
– Outpatient - do NOT code pneumonia. Code
to the highest degree of certainty (often a
sign/symptom). i.e. cough, fever
– Inpatient - code pneumonia as if it existed
because the evaluation and management of
the suspected condition is equal to the
treatment of the same condition that has been
confirmed
241
Ruled Out
• Ruled Out designates the fact that the
condition does NOT exist and should not
b coded.
be
d d
– Code the preceding signs, symptoms or
abnormal test results instead.
242
CCS Prep Workshop
Kim Felix, RHIA, CCS
AHIMA Approved ICD-10 Trainer
121
IOD Incorporated
May 2013
Sample Question
• A patient with a complaint of cough(786.2) was
referred by his physician to the outpatient
department for a chest x-ray (V72.5) to rule out
pneumonia (486).
(486) The results were negative.
negative
Which of the following is the appropriate
sequencing?
A. V72.5; 786.2
B. 786.2
C. V72.5; 486
D. V72.5
243
Answer
•
B. 786.2
244
CCS Prep Workshop
Kim Felix, RHIA, CCS
AHIMA Approved ICD-10 Trainer
122
IOD Incorporated
May 2013
Cancelled procedures
• If the original treatment plan cannot be
carried out due to unforeseen
circumstances the criteria for designation
circumstances,
of principal diagnosis does not change.
• Example: Pt with BPH admitted for TURP.
After admission, the patient fell and
sustained a fracture of the femur. TURP
was canceled
canceled, hip pinning done the
following day. Principal dx: BPH, even
though it was not treated, it was the reason
for the admission.
Sample question
• The patient with a history of esophageal reflux is prepared
for an EGD. While infusing Fentanyl and Versed for
sedation, the patient developed sinus bradycardia and the
physician cancelled the procedure.
Assign the diagnosis code(s).
Select THREE of the following options
a.
b.
c.
d.
e.
427.81 (sinoatrial node dysfunction)
427.89 (other cardiac dysrhythmias)
530.11 (reflux esophagitis)
530.81 (GERD)
V64.1 (surgery not carried out because of
contraindication)
f. V64.3 (surgery not carried out for other
reasons)
CCS Prep Workshop
Kim Felix, RHIA, CCS
AHIMA Approved ICD-10 Trainer
123
IOD Incorporated
May 2013
Answer
b. 427.89 (other cardiac dysrhythmias)
d. 530.81 (GERD)
f. V64.3 (surgery not carried out for other
reasons)
All those acronyms
• DSM = Psych/Behavioral health
• ICD-O = Oncology/Neoplasms
• SNOMED = Systemized Nomenclature of
M di i
Medicine
• SNOP = Standard Nomenclature of Pathology
• LOINC = Logical Observation Identifiers Names
and Codes (labs)
• AHA = Coding Clinic
• AMA = CPT
• ACOS = Cancer program
• Level II HCPCS = Medicare/Medicaid
248
CCS Prep Workshop
Kim Felix, RHIA, CCS
AHIMA Approved ICD-10 Trainer
124
IOD Incorporated
May 2013
More acronyms
•
•
•
•
•
•
•
CMI= Case Mix Index
RW = Relative weight
OASIS = PPS for home health
APC = PPS for outpatient
DRG = PPS for inpatient
QIO = Peer review (PEPPER reports)
RBRVs = Payment to MD for outpatient
surgery
• OIG Workplan
• NCHS = ICD-9-CM
249
Case Mix Index
#of
patients
DRG
10
20
10
5
222
232
451
530
Relative
Weight
3.00
2.00
2.00
1.00
Calculations
10 x 3.00= 30
20 x 2.00= 40
10 x 2.00= 20
5 x 1.00= 5
• MULTIPLY # patients by relative weight for each DRG in aqua
• ADD Relative weights (30+ 40+ 20+ 5 = 95)
• ADD # of patients (10+ 20+ 10+ 5= 45 )
• DIVIDE Total Relative Weights by # of patients 95 (total RW) ÷ 45
(# patients) = 2.111
2.111 is the CMI
250
CCS Prep Workshop
Kim Felix, RHIA, CCS
AHIMA Approved ICD-10 Trainer
125
IOD Incorporated
May 2013
Medical Science
Technology/Pharmacology
•
•
•
•
Anatomy and Physiology
Causes of disease processes
Diagnostic tests performed to work up
Treatment/Medication for specific
diagnoses
251
Physician Queries
• When is a query appropriate?
– Information in the record is conflicting, ambiguous
or incomplete regarding SIGNIFICANT reportable
conditions or procedures
• When is a query not necessary?
– If there is no conflicting documentation from one
physician to another
– When a physician has documented a final diagnosis
and clinical indicators do not appear to support this
diagnosisg
this is a p
physician’s
y
medical jjudgment
g
252
CCS Prep Workshop
Kim Felix, RHIA, CCS
AHIMA Approved ICD-10 Trainer
126
IOD Incorporated
May 2013
Recommended “practice exercises”
• Professional Review Guide for the CCS
examination (PRG Publishing, Inc)
– CD-ROM
CD ROM is
i like
lik ttaking
ki th
the actual
t l CCS exam
• Clinical Coding Workout (AHIMA)
253
Websites
• CCS examination details
– www.ahima.org
• POAs
– http://www.cms.hhs.gov/HospitalAcqCond
• Updated coding guidelines
– http://www.cms.gov/ICD9ProviderDiagnostic
Codes/07 summarytables asp
Codes/07_summarytables.asp
254
CCS Prep Workshop
Kim Felix, RHIA, CCS
AHIMA Approved ICD-10 Trainer
127
IOD Incorporated
May 2013
Thank You & Good Luck!
Kim Felix, RHIA, CCS, AHIMA-Approved ICD-10 Trainer
Director of Education
For further information on IOD and
how we can help you:
www.iodincorporated.com
255
CCS Prep Workshop
Kim Felix, RHIA, CCS
AHIMA Approved ICD-10 Trainer
128